Med Sure Exam 2

Lakukan tugas rumah & ujian kamu dengan baik sekarang menggunakan Quizwiz!

a client with CHF took cold medicine for her flu. she presents with new productive cough, pink frothy sputum, and worsening crackles. What action should the nurse take first? 1. assesslung sounds 2. give bumetanide IV push 3. notify HCP 4. clock out for lunch

2 = give bumetanide are in extreme HF, meaning heavy fluid giving IDE drug / diuretics to dry out fluid

A nurse teaches a client who is prescribed digoxin therapy. Which statement would the nurse include in this client's teaching? a. "Avoid taking aspirin or aspirin-containing products." b. "Increase your intake of foods that are high in potassium." c. "Hold this medication if your pulse rate is below 80 beats/min." d. "Do not take this medication within 1 hour of taking an antacid."

AND = D Gastrointestinal absorption of digoxin is erratic. Many medications, especially antacids, interfere with its absorption. Clients are taught to hold their digoxin for bradycardia; a heart rate of 80 beats/min is too high for this cutoff. Potassium and aspirin have no impact on digoxin absorption.

A nurse plans care for a client who is experiencing dyspnea and must stop multiple times when climbing a flight of stairs. Which intervention would the nurse include in this client's plan of care? a. Assistance with activities of daily living b. Physical therapy activities every day c. Oxygen therapy at 2 L per nasal cannula d. Complete bedrest with frequent repositioning

ANS: A A client with dyspnea and the inability to complete activities such as climbing a flight of stairs without pausing has class IV dyspnea. The nurse would provide assistance with activities of daily living. These clients would be encouraged to participate in activities as tolerated. They would not be on complete bedrest, may not be able to tolerate daily physical therapy, and only need oxygen if hypoxia is present.

A nurse is caring for a client with a nonhealing arterial lower leg ulcer. What action by the nurse is best? a. Consult with the wound care nurse. b. Give pain medication prior to dressing changes. c. Maintain sterile technique for dressing changes. d. Prepare the client for eventual amputation.

ANS: A A nonhealing wound needs the expertise of the wound care nurse. Premedicating prior to painful procedures and maintaining sterile technique are helpful, but if the wound is not healing, more needs to be done. The client may need an amputation, but other options need to be tried first.

A nurse is interested in providing community education and screening on hypertension. In order to reach a priority population, to what target audience would the nurse provide this service? a. African-American churches b. Asian-American groceries c. High school sports camps d. Women's health clinics

ANS: A African Americans in the United States have one of the highest rates of hypertension in the world. The nurse has the potential to reach this priority population by providing services at African-American churches. Although hypertension education and screening are important for all groups, African Americans are the priority population for this intervention.

A nurse is teaching a female client about alcohol intake and how it affects hypertension. The client asks if drinking two beers a night is an acceptable intake. What answer by the nurse is best? a. "No, women should only have one beer a day as a general rule." b. "No, you should not drink any alcohol with hypertension." c. "Yes, since you are larger, you can have more alcohol." d. "Yes, two beers per day is an acceptable amount of alcohol."

ANS: A Alcohol intake should be limited to two drinks a day for men and one drink a day for women. A "drink" is classified as one beer, 1.5 ounces of hard liquor, or 5 ounces of wine. Limited alcohol intake is acceptable with hypertension. The woman's size does not matter.

A nurse admits a client from the emergency department. Client data are listed below: History ° 70 years of age ° History of diabetes ° On insulin twice a day ° Reports new onset dyspnea and productive cough Physical Assessment ° Crackles and rhonchi heard throughout the lungs ° Dullness to percussion ° Afebrile ° Oriented to person only Laboratory Values ° WBC 5,200/mm3 (5.2 x 10^9/L) ° PaO2 on room air 85 mm Hg What action by the nurse is the priority? a. Administer oxygen at 4 L per nasal cannula. b. Begin broad-spectrum antibiotics. c. Collect a sputum sample for culture. d. Start an IV of normal saline at 50 mL/hr.

ANS: A All actions are appropriate for this client who has signs and symptoms of pneumonia. However, airway and breathing come first, so begin oxygen administration and titrate it to maintain saturations greater than 95%. Start the IV and collect a sputum culture, and then begin antibiotics.

A nurse is caring for a client who is intubated and has an intra-aortic balloon pump. The client is restless and agitated. What action would the nurse perform first for comfort? a. Allow family members to remain at the bedside. b. Ask the family if the client would like a fan in the room. c. Keep the television tuned to the client's favorite channel. d. Speak loudly to the client in case of hearing problems.

ANS: A Allowing the family to remain at the bedside can help calm the client with familiar voices (and faces if the client wakes up). A fan might be helpful but may also spread germs through air movement or may agitate the client further. The TV would not be kept on all the time to allow for rest. Speaking loudly may agitate the client more.

A client has been diagnosed with a very large pulmonary embolism (PE) and has a dropping blood pressure. What medication would the nurse being most beneficial? a. Alteplase b. Enoxaparin c. Unfractionated heparin d. Warfarin sodium

ANS: A Alteplase is a "clot-busting" agent indicated in large PEs in the setting of hemodynamic instability. The nurse knows that this drug is the priority, although heparin may be started initially. Enoxaparin and warfarin are not indicated in this setting.

A nurse is teaching a client with heart failure who has been prescribed enalapril. Which statement would the nurse include in this client's teaching? a. "Avoid using salt substitutes." b. "Take your medication with food." c. "Avoid using aspirin-containing products." d. "Check your pulse daily."

ANS: A Angiotensin-converting enzyme (ACE) inhibitors such as enalapril inhibit the excretion of potassium. Hyperkalemia can be a life-threatening side effect, and clients should be taught to limit potassium intake. Salt substitutes are composed of potassium chloride. ACE inhibitors do not need to be taken with food and have no impact on the client's pulse rate. Aspirin is often prescribed in conjunction with ACE inhibitors and is not contraindicated.

A nurse is assessing a client with peripheral artery disease (PAD). The client states that walking five blocks is possible without pain. What question asked next by the nurse will give the best information? a. "Could you walk further than that a few months ago?" b. "Do you walk mostly uphill, downhill, or on flat surfaces?" c. "Have you ever considered swimming instead of walking?" d. "How much pain medication do you take each day?"

ANS: A As PAD progresses, it takes less oxygen demand to cause pain. Needing to cut down on activity to be pain free indicates that the client's disease is worsening. The other questions are useful, but not as important.

A nurse admits a client who is experiencing an exacerbation of heart failure. What action would the nurse take first? a. Assess the client's respiratory status. b. Draw blood to assess the client's serum electrolytes. c. Administer intravenous furosemide. d. Ask the client about current medications.

ANS: A Assessment of respiratory and oxygenation status is the most important nursing intervention for the prevention of complications. Monitoring electrolytes, administering diuretics, and asking about current medications are important but do not take precedence over assessing respiratory status.

A nurse auscultates a harsh hollow sound over a client's trachea and larynx. What action would the nurse take first? a. Document the findings. b. Administer oxygen therapy. c. Position the client in high-Fowler position. d. Administer prescribed albuterol.

ANS: A Bronchial breath sounds, including harsh, hollow, tubular, and blowing sounds, are a normal finding over the trachea and larynx. The nurse would document this finding. There is no need to implement oxygen therapy, administer albuterol, or change the client's position because the finding is normal.

1A nurse assesses clients on a cardiac unit. Which client would the nurse identify as being at greatest risk for the development of left-sided heart failure? a. A 36-year-old woman with aortic stenosis b. A 42-year-old man with pulmonary hypertension c. A 59-year-old woman who smokes cigarettes daily d. A 70-year-old man who had a cerebral vascular accident

ANS: A Causes of left ventricular failure include mitral or aortic valve disease, coronary artery disease, and hypertension. Pulmonary hypertension and chronic cigarette smoking are risk factors for right ventricular failure. A cerebral vascular accident does not increase the risk of heart failure.

A nurse assesses a client with tachycardia. Which clinical manifestation requires immediate intervention by the nurse? a. Midsternal chest pain b. Increased urine output c. Mild orthostatic hypotension d. P wave touching the T wave

ANS: A Chest pain, possibly angina, indicates that tachycardia may be increasing the client's myocardial workload and oxygen demand to such an extent that normal oxygen delivery cannot keep pace. This results in myocardial hypoxia and pain. Increased urinary output and mild orthostatic hypotension are not life-threatening conditions and therefore do not require immediate intervention. The P wave touching the T wave indicates significant tachycardia and would be assessed to determine the underlying rhythm and cause; this is an important assessment but is not as critical as chest pain, which indicates cardiac cell death.

A nurse is caring for a client who had a myocardial infarction. The nurse is confused because the client states that nothing is wrong and yet listens attentively while the nurse provides education on lifestyle changes and healthy menu choices. What response by the charge nurse is best? a. "Continue to educate the client on possible healthy changes." b. "Emphasize complications that can occur with noncompliance." c. "Tell the client that denial is normal and will soon go away." d. "You need to make sure the client understands this illness."

ANS: A Clients are often in denial after a coronary event. The client who seems to be in denial but is compliant with treatment may be using a healthy form of coping that allows time to process the event and start to use problem-focused coping. The nurse would not discourage this type of denial and coping, but rather continue providing education in a positive manner. Emphasizing complications may make the client defensive and more anxious. Telling the client that denial is normal is placing too much attention on the process. Forcing the client to verbalize understanding of the illness is also potentially threatening to the client.

A client has peripheral arterial disease (PAD). What statement by the client indicates misunderstanding about self-management activities? a. "I can use a heating pad on my legs if it's set on low." b. "I should not cross my legs when sitting or lying down." c. "I will go out and buy some warm, heavy socks to wear." d. "It's going to be really hard but I will stop smoking."

ANS: A Clients with PAD should never use heating pads as skin sensitivity is diminished and burns can result. The other statements show good understanding of self-management.

A client is in the clinic a month after having a myocardial infarction. The client reports sleeping well since moving into the guest bedroom. What response by the nurse is best? a. "Do you have any concerns about sexuality?" b. "I'm glad to hear you are sleeping well now." c. "Sleep near your spouse in case of emergency." d. "Why would you move into the guest room?"

ANS: A Concerns about resuming sexual activity are common after cardiac events. The nurse would gently inquire if this is the issue. While it is good that the client is sleeping well, the nurse would investigate the reason for the move. The other two responses are likely to cause the client to be defensive.

A nurse cares for a client with right-sided heart failure. The client asks, "Why do I need to weigh myself every day?" How would the nurse respond? a. "Weight is the best indication that you are gaining or losing fluid." b. "Daily weights will help us make sure that you're eating properly." c. "The hospital requires that all clients be weighed daily." d. "You need to lose weight to decrease the incidence of heart failure."

ANS: A Daily weights are needed to document fluid retention or fluid loss. One liter of fluid equals 2.2 lb (1 kg). Weight changes are the most reliable indicator of fluid loss or gain. The other responses do not address the importance of monitoring fluid retention or loss.

A nurse cares for an older adult client with heart failure. The client states, "I don't know what to do. I don't want to be a burden to my daughter, but I can't do it alone. Maybe I should die." What is the best response by the nurse? a. "I can stay if you would you like to talk more about this." b. "You are lucky to have such a devoted daughter." c. "It is normal to feel as though you are a burden." d. "Would you like to meet with the chaplain?"

ANS: A Depression can occur in clients with heart failure, especially older adults. Having the client talk about his or her feelings will help the nurse focus on the actual problem. Open-ended statements allow the client to respond safely and honestly. The other options minimize the client's concerns and do not allow the nurse to obtain more information to provide client-centered care.

A client is to receive a dopamine infusion. What does the nurse do to prepare for this infusion? a. Gather central line supplies. b. Mark the client's pedal pulses. c. Monitor the client's vital signs. d. Ensure an accurate weight is charted.

ANS: A Dopamine should be infused through a central line to prevent extravasation and necrosis of tissue. The nurse would gather supplies for the primary health care provider to insert a central line. Monitoring vital signs is important for any client who has an acute cardiac problem, but this doesn't give the frequency of evaluation. Marking the client's pedal pulses and ensuring a weight is documented are not related to this infusion.

An emergency department nurse obtains the health history of a client. Which statement by the client would alert the nurse to the occurrence of heart failure? a. "I get short of breath when I climb stairs." b. "I see halos floating around my head." c. "I have trouble remembering things." d. "I have lost weight over the past month."

ANS: A Dyspnea on exertion is an early manifestation of heart failure and is associated with an activity such as stair climbing. The other findings are not specific to early occurrence of heart failure.

While assessing a client who has facial trauma, the nurse auscultates stridor. The client is anxious and restless. What action would the nurse take first? a. Contact the primary health care provider and prepare for intubation. b. Administer prescribed albuterol nebulizer therapy. c. Place the client in high-Fowler position. d. Ask the client to perform deep-breathing exercises.

ANS: A Facial and neck tissue edema can occur in clients with facial trauma. Airway patency is the highest priority. Clients who experience stridor and hypoxia, manifested by anxiety and restlessness, would be immediately intubated to ensure airway patency. Albuterol decreases bronchi and bronchiole inflammation, not facial and neck edema. Although putting the client in high-Fowler position and asking the client to perform breathing exercises may temporarily improve the patient's comfort, these actions will not decrease the underlying problem or improve airway patency.

The nurse is caring for four hypertensive clients. Which drug-laboratory value combination would the nurse report immediately to the health care provider? a. Furosemide/potassium: 2.1 mEq/L b. Hydrochlorothiazide/potassium: 4.2 mEq/L c. Spironolactone/potassium: 5.1 mEq/L d. Torsemide/sodium: 142 mEq/L

ANS: A Furosemide is a loop diuretic and can cause hypokalemia. A potassium level of 2.1 mEq/L is quite low and would be reported immediately. Spironolactone is a potassium-sparing diuretic that can cause hyperkalemia. A potassium level of 5.1 mEq/L is on the high side, but it is not as critical as the low potassium with furosemide. The other two laboratory values are normal.

The emergency department (ED) manager is reviewing client charts to determine how well the staff perform when treating clients with community-acquired pneumonia. What outcome demonstrates that goals for this client type have been met? a. Antibiotics started before admission. b. Blood cultures obtained within 20 minutes. c. Chest x-ray obtained within 30 minutes. d. Pulse oximetry obtained on all clients.

ANS: A Goals for treatment of community-acquired pneumonia include initiating antibiotics prior to inclient admission or within 6 hours of presentation to the ED. Timely collection of blood cultures, chest x-ray, and pulse oximetry are important as well but do not coincide with established goals.

A client had a femoral-popliteal bypass graft with a synthetic graft. What action by the nurse is most important to prevent wound infection? a. Appropriate hand hygiene before giving care b. Assessing the client's temperature every 4 hours c. Clean technique when changing dressings d. Monitoring the client's daily white blood cell count

ANS: A Hand hygiene is the best way to prevent infections in hospitalized clients. Dressing changes would be done with sterile technique. Assessing vital signs and white blood cell count will not prevent infection.

A client had a percutaneous angioplasty for renovascular hypertension 3 months ago. What assessment finding by the nurse indicates that an important outcome for this client has been met? a. Client is able to decrease blood pressure medications. b. Insertion site has healed without redness or tenderness. c. Most recent lab data show BUN: 19 mg/dL and creatinine 1.1 mg/dL. d. Verbalizes understanding of postprocedure lifestyle changes.

ANS: A Hypertension can be caused by renovascular disease. Opening up a constricted renal artery can lead to decreased blood pressure, manifested by the need for less blood pressure medication. The other findings are normal and desired, but not specifically related to hypertension caused by renal disease.

A nurse cares for a client who has hypertension that has not responded well to several medications. The client states compliance is not an issue. What action would the nurse take next? a. Assess the client for obstructive sleep apnea. b. Arrange a home sleep apnea test. c. Encourage the client to begin exercising. d. Schedule a polysomnography

ANS: A Hypertension not responding to medications can be a sign of obstructive sleep apnea (OSA). The nurse would assess the client using an evidence-based tool, such as the STOP-Bang Sleep Apnea Questionnaire, the Epworth Sleepiness Scale, the Pittsburgh Sleep Quality Index, and the Multiple Sleep Latency Test. If the results of the assessment indicate OSA may be a problem, the nurse would consult the primary health care provider for further testing. An at-home sleep-study is often done prior to a polysomnography. Excessive weight can contribute to OSA so exercising is always encouraged, but this is not specific to assessing for OSA.

A client is taking warfarin and asks the nurse if taking St. John's wort is acceptable. What response by the nurse is best? a. "No, it may interfere with the warfarin." b. "There isn't any information about that." c. "Why would you want to take that?" d. "Yes, it is a good supplement for you."

ANS: A Many foods and drugs interfere with warfarin, St. John's wort being one of them. The nurse would advise the client against taking it. The other answers are not accurate.

An older adult is on cardiac monitoring after a myocardial infarction. The client shows frequent dysrhythmias. What action by the nurse is most appropriate? a. Assess for any hemodynamic effects of the rhythm. b. Prepare to administer antidysrhythmic medication. c. Notify the primary health care provider or call the Rapid Response Team. d. Turn the alarms off on the cardiac monitor.

ANS: A Older clients may have dysrhythmias due to age-related changes in the cardiac conduction system. Or this client's dysrhythmias could be a consequence of the myocardial infarction. They may or may not have significant hemodynamic effects. The nurse would first assess for the effects of the dysrhythmia before proceeding further. The alarms on a cardiac monitor would never be shut off. The other two actions may or may not be needed.

A client is receiving oxygen at 4 L per nasal cannula. What comfort measure may the nurse delegate to assistive personnel (AP)? a. Apply water-soluble ointment to nares and lips. b. Periodically turn the oxygen down or off. c. Replaces the oxygen tubing with a different type. d. Turn the client every 2 hours or as needed.

ANS: A Oxygen can be drying, so the UAP can apply water-soluble lubricant to the client's lips and nares. The AP would not adjust the oxygen flow rate or replace the tubing. Turning the client is not related to comfort measures for oxygen.

A nurse teaches a client who experiences occasional premature atrial contractions (PACs) accompanied by palpitations that resolve spontaneously without treatment. Which statement would the nurse include in this client's teaching? a. "Minimize or abstain from caffeine." b. "Lie on your side until the attack subsides." c. "Use your oxygen when you experience PACs." d. "Take amiodarone daily to prevent PACs."

ANS: A PACs usually have no hemodynamic consequences. For a client experiencing infrequent PACs, the nurse would explore possible lifestyle causes, such as excessive caffeine intake and stress. Lying on the side will not prevent or resolve PACs. Oxygen is not necessary. Although medications may be needed to control symptomatic dysrhythmias, for infrequent PACs, the client first would try lifestyle changes to control them.

A client appears dyspneic, but the oxygen saturation is 97%. What action by the nurse is best? a. Assess for other signs of hypoxia. b. Change the sensor on the pulse oximeter. c. Obtain a new oximeter from central supply. d. Tell the client to take slow, deep breaths.

ANS: A Pulse oximetry is not always the most accurate assessment tool for hypoxia as many factors can interfere, producing normal or near-normal readings in the setting of hypoxia. The nurse would conduct a more thorough assessment. The other actions are not appropriate for a hypoxic client.

A nurse cares for a client with a 40-year smoking history who is experiencing distended neck veins and dependent edema. Which physiologic process would the nurse correlate with this client's history and clinical signs and symptoms? a. Increased pulmonary pressure creating a higher workload on the right side of the heart b. Exposure to irritants resulting in increased inflammation of the bronchi and bronchioles c. Increased number and size of mucous glands producing large amounts of thick mucus d. Left ventricular hypertrophy creating a decrease in cardiac output

ANS: A Smoking increases pulmonary hypertension, resulting in cor pulmonale, or right-sided heart failure. Increased pressures in the lungs make it more difficult for blood to flow through the lungs. Blood backs up into the right side of the heart and then into the peripheral venous system, creating distended neck veins and dependent edema. Inflammation in bronchi and bronchioles creates an airway obstruction which manifests as wheezes. Thick mucus in the lungs has no impact on distended neck veins and edema. Left ventricular hypertrophy is associated with left-heart failure and is not directly caused by a 40-year smoking history.

A nurse is demonstrating suctioning a tracheostomy during the annual skills review. What action by the student demonstrates that more teaching is needed? a. Applying suction while inserting the catheter b. Preoxygenating the client prior to suctioning c. Suctioning for a total of three times if needed d. Suctioning for only 10 to 15 seconds each time

ANS: A Suction would only be applied while withdrawing the catheter. The other actions are appropriate.

A client admitted for pneumonia has been tachypneic for several days. When the nurse starts an IV to give fluids, the client questions this action, saying "I have been drinking tons of water. How am I dehydrated?" What response by the nurse is best? a. "Breathing so quickly can be dehydrating." b. "Everyone with pneumonia is dehydrated." c. "This is really just to administer your antibiotics." d. "Why do you think you are so dehydrated?"

ANS: A Tachypnea and mouth breathing (from increased work of breathing), both seen in pneumonia, increase insensible water loss and can lead to a degree of dehydration. The other options do not give the client useful information that addresses this specific concern.

A nurse assesses a client who has aortic regurgitation. In which location in the illustration shown below would the nurse auscultate to best hear a cardiac murmur related to aortic regurgitation? a. Location A: aortic b. Location B: pulmonic c. Location C: tricuspid d. Location D: mitral

ANS: A The aortic valve is auscultated in the second intercostal space just to the right of the sternum. The pulmonic valve would be heard in location B located in the second intercostal space just left of the sternum. The mitral valve would be heard in location D located in the fifth intercostal space at the apex of the heart. The tricuspid valve would be heard in location C located in the fifth intercostal space at the lower left of the sternal border.

An assistive personnel (AP) was feeding a client with a tracheostomy. Later that evening, the UAP reports that the client had a coughing spell during the meal. What action by the nurse is best? a. Assess the client's lung sounds. b. Assign a different AP to the client. c. Report the AP to the manager. d. Request thicker liquids for meals.

ANS: A The best action is to check the client's oxygenation because he or she may have aspirated. Once the client has been assessed, the nurse would notify the primary health care provider of possible aspiration and would consult with the registered dietitian about appropriately thickened liquids. The UAP should have reported the incident immediately, but addressing that issue is not the immediate priority.

A nurse cares for a client with infective endocarditis. Which infection control precautions would the nurse use? a. Standard Precautions b. Bleeding Precautions c. Reverse isolation d. Contact isolation

ANS: A The client with infective endocarditis does not pose any specific threat of transmitting the causative organism. Standard Precautions would be used. Bleeding Precautions, reverse isolation, or Contact Precautions are not necessary.

A nurse assesses a client who has a nasal fracture. The client reports constant nasal drainage, a headache, and difficulty with vision. What action would the nurse take next? a. Collect the nasal drainage on a piece of filter paper. b. Encourage the client to blow his or her nose. c. Perform a test focused on a neurologic examination. d. Palpate the nose, face, and neck.

ANS: A The client with nasal drainage after facial trauma could have a skull fracture resulting in leakage of cerebrospinal fluid (CSF). CSF can be differentiated from regular drainage by the fact that it forms a halo when dripped on filter paper and tests positive for glucose. The other actions would be appropriate but are not as high a priority as assessing for CSF. A CSF leak would increase the patient's risk for infection.

A nurse assesses a client who is prescribed fluticasone and notes oral lesions. What action would the nurse take? a. Encourage oral rinsing after fluticasone administration. b. Obtain an oral specimen for culture and sensitivity. c. Start the client on a broad-spectrum antibiotic. d. Document the finding as a known side effect.

ANS: A The drug reduces local immunity and increases the risk for local infection, especially Candida albicans. Rinsing the mouth after using the inhaler will decrease the risk for developing this infection. Use of mouthwash and broad-spectrum antibiotics is not warranted in this situation. The nurse would document the finding, but the best action to take is to have the client start rinsing his or her mouth after using fluticasone. An oral specimen for culture and sensitivity is not necessary to care for this client.

A client has been diagnosed with a deep vein thrombosis and is to be discharged on warfarin. The client is adamant about refusing the drug because "it's dangerous." What action by the nurse is best? a. Assess the reason behind the client's fear. b. Remind the client about laboratory monitoring. c. Tell the client that drugs are safer today than before. d. Warn the client about consequences of noncompliance.

ANS: A The first step is to assess the reason behind the client's fear, which may be related to the experience of someone the client knows who took warfarin or misinformation. If the nurse cannot address the specific rationale, teaching will likely be unsuccessful. Laboratory monitoring once every few weeks may not make the client perceive the drug to be safe. General statements like "drugs are safer today" do not address the root cause of the problem. Warning the client about possible consequences of not taking the drug is not therapeutic and is likely to lead to an adversarial relationship.

A nurse prepares to discharge a client with a cardiac dysrhythmia who is prescribed home health care services. Which priority information would be communicated to the home health nurse upon discharge? a. Medication orders for home b. Immunization history c. Religious beliefs d. Nutrition preferences

ANS: A The home health nurse needs to know current medications the client is taking to ensure assessment, evaluation, and further education related to these medications. The other information might be used to plan care, but not as the priority.

A nurse assesses a client who is recovering from a myocardial infarction. The client's blood pressure is 140/88 mm Hg. What action would the nurse take first? a. Compare the results with previous blood pressure readings. b. Increase the intravenous fluid rate because these readings are low. c. Immediately notify the primary health care provider of the elevated blood pressure. d. Document the finding in the client's chart as the only action.

ANS: A The most recent range for normal blood pressure is less than 140 mm Hg systolic and less than 90mm Hg diastolic. This client's blood pressure is at the upper range of acceptable, so the nurse would compare the client's current reading with those previously recorded before doing anything else. The reading is not low, so the nurse would not increase IV fluids, nor would the nurse necessarily notify the primary health care provider. Documentation is important, but the nurse first checks previous readings.

A nurse is caring for a client on mechanical ventilation and finds the client agitated and thrashing about. What action by the nurse is most appropriate? a. Assess the cause of the agitation. b. Reassure the client that he or she is safe. c. Restrain the client's hands. d. Sedate the client immediately.

ANS: A The nurse needs to determine the cause of the agitation. The inability to communicate often makes clients anxious, even to the point of panic. Pain, confusion, and hypoxia can also cause agitation. Once the nurse determines the cause of the agitation, he or she can implement measures to relieve the underlying cause. Reassurance is also important but may not address the etiology of the agitation. Restraints and more sedation may be necessary but not as a first step. Ensuring the client is adequately oxygenated is the priority.

A client has been hospitalized with tuberculosis (TB). The client's spouse is fearful of entering the room where the client is in isolation and refuses to visit. What action by the nurse is best? a. Ask the spouse to explain the fear of visiting in further detail. b. Inform the spouse that the precautions are meant to keep other clients safe. c. Show the spouse how to follow the Isolation Precautions to avoid illness. d. Tell the spouse that he or she has already been exposed, so it's safe to visit.

ANS: A The nurse needs to obtain further information about the spouse's specific fears so they can be addressed. This will decrease stress and permit visitation, which will be beneficial for both client and spouse. Precautions for TB prevent transmission to all who come into contact with the client. Explaining Isolation Precautions and what to do when entering the room will be helpful, but this is too narrow in scope to be the best answer. Telling the spouse that it's safe to visit is demeaning of the spouse's feelings.

While assessing a client on a cardiac unit, a nurse identifies the presence of an S3 gallop. What action would the nurse take next? a. Assess for symptoms of left-sided heart failure. b. Document this as a normal finding. c. Call the primary health care provider immediately. d. Transfer the client to the intensive care unit.

ANS: A The presence of an S3 gallop is an early diastolic filling sound indicative of increasing left ventricular pressure and left ventricular failure. The other actions are not warranted.

A client has been diagnosed with tuberculosis (TB). What action by the nurse takes highest priority? a. Educating the client on adherence to the treatment regimen b. Encouraging the client to eat a well-balanced diet c. Informing the client about follow-up sputum cultures d. Teaching the client ways to balance rest with activity

ANS: A The treatment regimen for TB often ranges from 26 weeks, but can be up to 2 years, making adherence problematic for many people. The nurse would stress the absolute importance of following the treatment plan for the entire duration of prescribed therapy. The other options are appropriate topics to educate this client on but do not take priority.

A nurse is working with a client who takes clopidogrel. The client's recent laboratory results include a blood urea nitrogen (BUN) of 33 mg/dL and creatinine of 2.8 mg/dL. What action by the nurse is best? a. Ask if the client eats grapefruit. b. Assess the client for dehydration. c. Facilitate admission to the hospital. d. Obtain a random urinalysis.

ANS: A There is a drug-food interaction between clopidogrel and grapefruit that can lead to acute kidney failure. This client has elevated renal laboratory results, indicating some degree of kidney involvement. The nurse would assess if the client eats grapefruit or drinks grapefruit juice. Dehydration can cause the BUN to be elevated, but the elevation in creatinine is more specific for a kidney injury. The client does not necessarily need to be admitted. A urinalysis may or may not be ordered.

A client with a history of heart failure and hypertension is in the clinic for a follow-up visit. The client is on lisinopril and warfarin. The client reports new-onset cough. What action by the nurse is most appropriate? a. Assess the client's lung sounds and oxygenation. b. Instruct the client on another antihypertensive. c. Obtain a set of vital signs and document them. d. Remind the client that cough is a side effect of lisinopril.

ANS: A This client could be having an exacerbation of heart failure or experiencing a side effect of lisinopril (and other angiotensin-converting enzyme inhibitors). The nurse would assess the client's lung sounds and other signs of oxygenation first. The client may or may not need to switch antihypertensive medications. Vital signs and documentation are important, but the nurse would assess the respiratory system first. If the cough turns out to be a side effect, reminding the client is appropriate, but then more action needs to be taken.

A client has a tracheostomy that is 3 days old. Upon assessment, the nurse notes that the client's face is puffy and the eyelids are swollen. What action by the nurse takes best? a. Assess the client's oxygen saturation. b. Notify the Rapid Response Team. c. Oxygenate the client with a bag-valve-mask. d. Palpate the skin of the upper chest.

ANS: A This client may have subcutaneous emphysema, which is air that leaks into the tissues surrounding the tracheostomy. The nurse would first assess the client's oxygen saturation and other indicators of oxygenation. If the client is stable, the nurse can palpate the skin of the upper chest to feel for the air. If the client is unstable, the nurse calls the Rapid Response Team. Using a bag-valve-mask device may or may not be appropriate for the unstable client.

A nurse supervises an assistive personnel (AP) applying electrocardiographic monitoring. Which statement would the nurse provide to the AP related to this procedure? a. "Clean the skin and clip hairs if needed." b. "Add gel to the electrodes prior to applying them." c. "Place the electrodes on the posterior chest." d. "Turn off oxygen prior to monitoring the client."

ANS: A To ensure the best signal transmission, the skin would be clean and hairs clipped. Electrodes would be placed on the anterior chest, and no additional gel is needed. Oxygen has no impact on electrocardiographic monitoring.

A client in the emergency department has several broken ribs and reports severe pain. What care measure will best promote comfort? a. Prepare to assist with intercostal nerve block. b. Humidify the supplemental oxygen. c. Splint the chest with a large ACE wrap. d. Provide warmed blankets and warmed IV fluids.

ANS: A Uncomplicated rib fractures generally are simple to manage; however, opioids may be needed for pain. For severe pain, an intercostal nerve block is beneficial. The client needs to be able to breathe deeply and cough so as not to get atelectasis and/or pneumonia. Humidifying the oxygen will not help with the pain. Rib fractures are not wrapped or splinted in any way because this inhibits chest movement. Warmed blankets and warm IV fluids are nice comfort measures, but do not help with severe pain.

A nurse assesses a client who had a myocardial infarction and has a blood pressure of 88/58 mm Hg. Which additional assessment finding would the nurse expect? a. Heart rate of 120 beats/min b. Cool, clammy skin c. Oxygen saturation of 90% d. Respiratory rate of 8 breaths/min

ANS: A When a client experiences hypotension, baroreceptors in the aortic arch sense a pressure decrease in the vessels. The parasympathetic system responds by lessening the inhibitory effect on the sinoatrial node. This results in an increase in heart rate and respiratory rate. This tachycardia is an early response and is seen even when blood pressure is not critically low. An increased heart rate and respiratory rate will compensate for the low blood pressure and maintain oxygen saturation and perfusion. The client may not be able to compensate for long and decreased oxygenation and cool, clammy skin will occur later.

A nurse is caring for a client with a history of renal insufficiency who is scheduled for a cardiac catheterization. What actions would the nurse take prior to the catheterization? (Select all that apply.) a. Assess for allergies to iodine. b. Administer intravenous fluids. c. Assess blood urea nitrogen (BUN) and creatinine results. d. Insert a Foley catheter. e. Administer a prophylactic antibiotic. f. Insert a central venous catheter.

ANS: A, B, C If the client has kidney disease, fluids may be given 12 to 24 hours before the procedure for renal protection. Hydration would continue after the procedure. The client would be assessed for allergies to iodine, including shellfish; the contrast medium used during the catheterization contains iodine. Baseline renal labs would be assessed. A Foley catheter and central venous catheter are not required for the procedure and would only increase the client's risk for infection. Prophylactic antibiotics are not administered prior to a cardiac catheterization.

A home health nurse is visiting a new client who uses oxygen in the home. For which factors does the nurse assess when determining if the client is using the oxygen safely? (Select all that apply.) a. The client does not allow smoking in the house. b. Electrical cords are in good working order. c. Flammable liquids are stored in the garage. d. Household light bulbs are the fluorescent type. e. The client does not have pets inside the home. f. No alcohol-based hand sanitizers are present.

ANS: A, B, C Oxygen it enhances combustion, so precautions are needed whenever using it. The nurse would assess if the client allows smoking in the house, whether electrical cords are in good shape or are frayed, and if flammable liquids are stored (and used) in the garage away from the oxygen. Light bulbs and pets are not related to oxygen safety. Alcohol-based hand sanitizers are permitted.

A hospital nurse is participating in a drill during which many "clients" with inhalation anthrax are being admitted. What drugs would the nurse anticipate administering? (Select all that apply.) a. Vancomycin b. Ciprofloxacin c. Doxycycline d. Ethambutol e. Sulfamethoxazole-trimethoprim (SMX-TMP)

ANS: A, B, C Vancomycin, ciprofloxacin, and doxycycline are all possible treatments for inhalation anthrax. Ethambutol is used for tuberculosis. SMX-TMP is commonly used for urinary tract infections and other common infections.

A nurse is caring for a client with a nonhealing arterial ulcer. The primary health care provider has informed the client about possibly needing to amputate the client's leg. The client is crying and upset. What actions by the nurse are best? (Select all that apply.) a. Ask the client to describe his or her current emotions. b. Assess the client for support systems and family. c. Offer to stay with the client if he or she desires. d. Relate how smoking contributed to this situation. e. Tell the client that many people have amputations. f. Arrange for an amputee to come visit the client.

ANS: A, B, C When a client is upset, the nurse would offer self by remaining with the client if desired. Other helpful measures include determining what and whom the client has for support systems and asking the client to describe what he or she is feeling. Telling the client how smoking has led to this situation will only upset the client further and will damage the therapeutic relationship. Telling the client that many people have amputations belittles the client's feelings. It is too early to send an amputee to visit the client as the decision to amputate has not yet been made.

A nurse teaches a client who is being discharged after a jaw wiring for a mandibular fracture. Which statements would the nurse include in this patient's teaching? (Select all that apply.) a. "You will need to cut the wires if you start vomiting." b. "Eat six soft or liquid meals each day while recovering." c. "Use a Waterpik for dental hygiene until you can brush again. d. "Sleep in a semi-Fowler position after the surgery." e. "Gargle with mouthwash that contains hydrogen peroxide once a day."

ANS: A, B, C, D The client needs to know how to cut the wires in case of emergency. If the client vomits, he or she may aspirate. The client would also be taught to eat soft or liquid meals multiple times a day, irrigate the mouth with a Waterpik to prevent infection, and sleep in a semi-Fowler position to assist in avoiding aspiration. Mouthwash with hydrogen peroxide is not a recommendation.

A client has been diagnosed with an empyema. What interventions would the nurse anticipate providing to this client? (Select all that apply.) a. Assisting with chest tube insertion b. Facilitating pleural fluid sampling c. Performing frequent respiratory assessment d. Providing antipyretics as needed e. Suctioning deeply every 4 hours

ANS: A, B, C, D The client with an empyema is often treated with chest tube insertion, which facilitates obtaining samples of the pleural fluid for analysis and re-expands the lungs. The nurse would perform frequent respiratory system assessments. Antipyretic medications are also used. Suction is only used when needed and is not done deeply to prevent tissue injury.

A nurse assesses a client who is 6 hours postsurgery for a nasal fracture and has nasal packing in place. What actions would the nurse take? (Select all that apply.) a. Observe for clear drainage. b. Assess for signs of bleeding. c. Watch the client for frequent swallowing. d. Ask the client to open his or her mouth. e. Administer a nasal steroid to decrease edema. f. Change the nasal packing.

ANS: A, B, C, D The nurse would observe for clear drainage because of the risk for cerebrospinal fluid leakage. The nurse would assess for signs of bleeding by asking the client to open his or her mouth and observing the back of the throat for bleeding. The nurse would also note whether the client is swallowing frequently because this could indicate postnasal bleeding. A nasal steroid would increase the risk for infection. It is too soon to change the packing, which would be changed by the surgeon the first time.

A nurse is assessing a client's history of particular matter exposure. What questions are consistent with the I PREPARE tool? (Select all that apply.) a. Investigate all history of known exposures. b. Determine if breathing problems are worse at work. c. Ask the client what type of heating is in the home. d. Gather details about the geographic location of the client's home. e. Have client list all previous jobs and work experiences. f. Assess what hobbies the client and family enjoy.

ANS: A, B, C, D, E, F All questions are appropriate for the I PREPARE model of particulate matter exposure. The R and final E stands for resources/referrals and educate.

The nurse caring for mechanically ventilated clients uses best practices to prevent ventilator-associated pneumonia. What actions are included in this practice? (Select all that apply.) a. Adherence to proper hand hygiene b. Administering antiulcer medication c. Elevating the head of the bed d. Providing oral care per protocol e. Suctioning the client on a regular schedule f. Turning and positioning the client at least every 2 hours

ANS: A, B, C, D, F The "ventilator bundle" is a group of care measures to prevent ventilator-associated pneumonia. Actions in the bundle include using proper hand hygiene, giving antiulcer medications, elevating the head of the bed, providing frequent oral care per policy, preventing aspiration, turning and positioning, and providing pulmonary hygiene measures. Suctioning is done as needed.

A client with a new pulmonary embolism (PE) is anxious. What nursing actions are most appropriate? (Select all that apply.) a. Acknowledge the frightening nature of the illness. b. Delegate a back rub to the assistive personnel (AP). c. Give simple explanations of what is happening. d. Request a prescription for antianxiety medication. e. Stay with the client and speak in a quiet, calm voice.

ANS: A, B, C, E Clients with PEs are often anxious. The nurse can acknowledge the client's fears, delegate comfort measures, give simple explanations the client will understand, and stay with the client. Using a calm, quiet voice is also reassuring. Sedatives and antianxiety medications are not used routinely because they can contribute to hypoxia. If the client's anxiety is interfering with diagnostic testing or treatment, they can be used, but there is no evidence that this is the case.

A nurse is teaching a client about possible complications and hazards of home oxygen therapy. About which complications does the nurse plan to teach the client? (Select all that apply.) a. Absorptive atelectasis b. Combustion c. Dried mucous membranes d. Alveolar recruitment e. Toxicity

ANS: A, B, C, E Complications of oxygen therapy include absorptive atelectasis, combustion, dried mucous membranes, and oxygen toxicity. Alveolar recruitment may be a benefit of high-flow nasal cannulas such as Vapotherm, which both humidifies and warms the oxygen.

A client, who has become increasingly dyspneic over a year, has been diagnosed with pulmonary fibrosis. What information would the nurse plan to include in teaching this client? (Select all that apply.) a. The need to avoid large crowds and people who are ill b. Safety measures to take if home oxygen is needed c. Information about appropriate use of the drug nintedanib d. Genetic therapy to stop the progression of the disease e. Measures to avoid fatigue during the day f. The possibility of receiving a lung transplant if infection-free for a year

ANS: A, B, C, E Pulmonary fibrosis is a progressive disorder with no cure. Therapy focuses on slowing progression and managing dyspnea. Clients need to avoid contracting infections so should be taught to stay away from large crowds and sick people. Home oxygen is needed and the nurse would teach safety measures related to oxygen. The drug nintedanib has shown to improve cellular regulation and slow progression of the disease. Gene therapy is not available. Energy conservation measures are also an important topic. Lung transplantation is an unlikely option due to selection criteria.

A nurse teaches a client who has chronic obstructive pulmonary disease. Which statements related to nutrition would the nurse include in this client's teaching? (Select all that apply.) a. "Avoid drinking fluids just before and during meals." b. "Rest before meals if you have dyspnea." c. "Have about six small meals a day." d. "Eat high-fiber foods to promote gastric emptying." e. "Use pursed-lip breathing during meals." f. "Choose soft, high-calorie, high-protein foods."

ANS: A, B, C, E, F Clients with COPD often are malnourished for several reasons. The nurse would teach the client not to drink fluids before and with meals to avoid early satiety. The client needs to rest before eating, and eat smaller frequent meals: 4 to 6 a day. Pursed-lip breathing will help control dyspnea. Food that is easy to eat will be less tiring and the client should choose high-calorie, high-protein foods.

A nurse assesses a client who is recovering from a heart transplant. Which assessment findings would alert the nurse to the possibility of heart transplant rejection? (Select all that apply.) a. Shortness of breath b. Abdominal bloating c. New-onset bradycardia d. Increased ejection fraction e. Hypertension f. Fatigue

ANS: A, B, C, F Clinical findings of heart transplant rejection include shortness of breath, fatigue, fluid gain, abdominal bloating, new-onset bradycardia, hypotension, atrial fibrillation or flutter, decreased activity tolerance, and decreased ejection fraction.

A nurse is teaching a client who has premature ectopic beats. Which education would the nurse include in this client's teaching? (Select all that apply.) a. Smoking cessation b. Stress reduction and management c. Avoiding vagal stimulation d. Adverse effects of medications e. Foods high in potassium f. Types of aerobic exercise

ANS: A, B, D A client who has premature beats or ectopic rhythms would be taught to stop smoking, manage stress, take medications as prescribed, and report adverse effects of medications. Clients with premature beats are not at risk for vasovagal attacks or potassium imbalances. While exercise is beneficial, aerobic exercise is not specifically linked to this client's educational needs.

A nurse assesses clients on a cardiac unit. Which clients would the nurse identify as at greatest risk for the development of acute pericarditis? (Select all that apply.) a. A 36-year-old woman with systemic lupus erythematosus (SLE) b. A 42-year-old man recovering from coronary artery bypass graft surgery c. A 59-year-old woman recovering from a hysterectomy d. An 80-year-old man with a bacterial infection of the respiratory tract e. An 88-year-old woman with a stage III sacral ulcer

ANS: A, B, D Acute pericarditis is most commonly associated with acute exacerbations of systemic connective tissue disease, including SLE; with Dressler syndrome, or inflammation of the cardiac sac after cardiac surgery or a myocardial infarction; and with infective organisms, including bacterial, viral, and fungal infections. Abdominal and reproductive surgeries and pressure injuries do not increase clients' risk for acute pericarditis.

The nurse caring for mechanically ventilated clients knows that older adults are at higher risk for weaning failure. What age-related changes contribute to this? (Select all that apply.) a. Chest wall stiffness b. Decreased muscle strength c. Inability to cooperate d. Less lung elasticity e. Poor vision and hearing f. Chronic anemia

ANS: A, B, D Age-related changes that increase the difficulty of weaning older adults from mechanical ventilation include increased stiffness of the chest wall, decreased muscle strength, and less elasticity of lung tissue. Not all older adults have an inability to cooperate or poor sensory acuity. Anemia can make it difficult to wean a client, but this is not a normal age-related change.

A nurse is caring for a client on IV infusion of heparin. What actions does this nurse include in the client's plan of care? (Select all that apply.) a. Assess the client for bleeding. b. Monitor the daily activated partial thromboplastin time (aPTT) results. c. Stop the IV for aPTT above baseline. d. Use an IV pump for the infusion. e. Weigh the client daily on the same scale.

ANS: A, B, D Assessing for bleeding, monitoring aPTT, and using an IV pump for the infusion are all important safety measures for heparin to prevent injury from bleeding. The aPTT needs to be 1.5 to 2.5 times normal in order to demonstrate that the heparin is therapeutic. Weighing the client is not related.

The nurse is reviewing risk factors in a client who has atherosclerosis. Which findings are most concerning? (Select all that apply.) a. Elevated low-density lipoprotein (LDL-C) b. Decreased levels of high-density lipoprotein cholesterol (HDL-C) c. Asian ethnicity d. History of smoking e. Blood pressure: 142/92 mm Hg on one occasion

ANS: A, B, D Elevated levels of lipids (fats) such as low-density lipoprotein cholesterol (LDL-C) and decreased levels of high-density lipoprotein cholesterol can cause chemical damage to blood vessel walls. Smoking can cause endothelial damage in addition to increasing a client's carbon monoxide levels. African American and Hispanic ethnicities carry an increased risk for atherosclerosis. Hypertension does increase atherosclerosis risk, but an elevated reading on one occasion is not classified as hypertension.

2. A client has been bedridden for several days after major abdominal surgery. What action does the nurse delegate to the assistive personnel (AP) for deep vein thrombosis (DVT) prevention? (Select all that apply.) a. Apply compression stockings. b. Assist with ambulation. c. Encourage coughing and deep breathing. d. Offer fluids frequently. e. Teach leg exercises.

ANS: A, B, D The AP can apply compression stockings, assist with ambulation, and offer fluids frequently to help prevent DVT. The AP can also encourage the client to do pulmonary exercises, but these do not decrease the risk of DVT. Teaching is a nursing function.

A nurse prepares to discharge a client who has heart failure. Which questions would the nurse ask to ensure this client's safety prior to discharging home? (Select all that apply.) a. "Are your bedroom and bathroom on the first floor?" b. "What social support do you have at home?" c. "Will you be able to afford your oxygen therapy?" d. "What spiritual beliefs may impact your recovery?" e. "Are you able to accurately weigh yourself at home?"

ANS: A, B, D To ensure safety upon discharge, the nurse would assess for structural barriers to functional ability, such as stairs. The nurse would also assess the client's available social support, which may include family, friends, and home health services. The client's beliefs about and ability to adhere to medication and treatments, including daily weights, would also be reviewed. The other questions do not specifically address the client's safety upon discharge.

While obtaining a client's health history, the client states, "I am allergic to avocados, molds, and grass." Which responses by the nurse are best? (Select all that apply.) a. "What happens when you are exposed to those things? b. "How do you treat these allergies?" c. "When was the last time you ate foods containing avocados?" d. "I will document this in your record so all so everyone knows." e. "Have you ever been in the hospital after an allergic response?" f. "How do manage to avoid grass and mold?"

ANS: A, B, D, E Nurses would assess clients who have allergies for the specific cause, treatment, and response to treatment. The nurse would also document the allergies in a prominent place in the client's medical record. Asking about the last time the client ate avocados does not provide any pertinent information for the client's plan of care. Asking how a client manages to avoid environmental allergies in this fashion also does not provide any pertinent information.

A nurse studying acute coronary syndromes learns that the pain of a myocardial infarction (MI) differs from stable angina in what ways? (Select all that apply.) a. Accompanied by shortness of breath b. Feelings of fear or anxiety c. Lasts less than 15 minutes d. No relief from taking nitroglycerin e. Pain occurs without known cause f. Can be precipitated by exertion or stress

ANS: A, B, D, E The pain from an MI is often accompanied by shortness of breath and fear or anxiety. It lasts longer than 15 minutes and is not relieved by nitroglycerin. It occurs without a known cause such as exertion or stress.

A nurse is caring for a client in acute respiratory failure who is on mechanical ventilation. What actions will promote comfort in this client? (Select all that apply.) a. Allow visitors at the client's bedside. b. Ensure that the client can communicate if awake. c. Keep the television tuned to a favorite channel. d. Provide back and hand massages when turning. e. Turn the client every 2 hours or more.

ANS: A, B, D, E There are many basic care measures that can be employed for the client who is on a ventilator. Allowing visitation, providing a means of communication, massaging the client's skin, and routinely turning and repositioning the client are some of them. Keeping the TV on will interfere with sleep and rest.

A nurse is planning discharge teaching on tracheostomy care for an older client. What factors does the nurse need to assess before teaching this particular client? (Select all that apply.) a. Cognition b. Dexterity c. Hydration d. Range of motion e. Vision f. Upper arm range of motion

ANS: A, B, D, E, F The older adult is at risk for having impairments in cognition, dexterity, range of motion, and vision that could limit the ability to perform tracheostomy care and would be assessed. Upper arm mobility is required to perform tracheostomy self-care. Hydration is not directly related to the ability to perform self-care.

A nurse plans care for a client who has chronic obstructive pulmonary disease and thick, tenacious secretions. Which interventions would the nurse include in this client's plan of care? (Select all that apply.) a. Ask the client to drink 2 L of fluids daily. b. Add humidity to the prescribed oxygen. c. Suction the client every 2 to 3 hours. d. Use a vibrating chest physiotherapy device. e. Encourage diaphragmatic breathing. f. Administer the ordered mucolytic agent.

ANS: A, B, D, F Interventions to decrease thick tenacious secretions include maintaining adequate hydration and providing humidified oxygen. These actions will help to thin secretions, making them easier to remove by coughing. The use of a vibrating chest physiotherapy device can also help clients remove thick secretions but is usually used in clients with cystic fibrosis. Mucolytic agents help thin secretions, making them easier to bring up. Although suctioning may assist with the removal of secretions, frequent suctioning can cause airway trauma and does not support the client's ability to successfully remove secretions through normal coughing. Diaphragmatic breathing is not used to improve the removal of thick secretions.

A nurse is assessing a client with left-sided heart failure. For which clinical manifestations would the nurse assess? (Select all that apply.) a. Pulmonary crackles b. Confusion c. Pulmonary hypertension d. Dependent edema e. Cough that worsens at night f. Jugular venous distention

ANS: A, B, E Left-sided heart failure occurs with a decrease in contractility of the heart or an increase in afterload. Most of the signs will be noted in the respiratory system. These include crackles, confusion (due to decreased oxygenation), and cough. Right ventricular failure is associated with pulmonary hypertension, edema, and jugular venous distention.

A nurse teaches a client with a new permanent pacemaker. Which instructions would the nurse include in this client's teaching? (Select all that apply.) a. "Until your incision is healed, do not submerge your pacemaker. Only take showers." b. "Report any pulse rates lower than your pacemaker settings." c. "If you feel weak, apply pressure over your generator." d. "Have your pacemaker turned off before having magnetic resonance imaging (MRI)." e. "Do not lift your left arm above the level of your shoulder for 8 weeks."

ANS: A, B, E The client would not submerge in water until the site has healed; after the incision is healed, the client may take showers or baths without concern for the pacemaker. The client would be instructed to report changes in heart rate or rhythm, such as rates lower than the pacemaker setting or greater than 100 beats/min. The client would be advised of restrictions on physical activity for 8 weeks to allow the pacemaker to settle in place. The client would never apply pressure over the generator and would avoid tight clothing. The client would never have MRI because, whether turned on or off, the pacemaker contains metal. The client would be advised to inform all health care providers that he or she has a pacemaker.

A nurse evaluates laboratory results for a client with heart failure. Which results would the nurse expect? (Select all that apply.) a. Hematocrit: 32.8% b. Serum sodium: 130 mEq/L (130 mmol/L) c. Serum potassium: 4.0 mEq/L (4.0 mmol/L) d. Serum creatinine: 1.0 mg/dL (88.4 mcmol/L) e. Proteinuria f. Microalbuminuria

ANS: A, B, E, F A hematocrit of 32.8% is low (should be 42.6%), indicating a dilutional ratio of red blood cells to fluid. A serum sodium of 130 mEq/L (130 mmol/L) is low because of hemodilution. Microalbuminuria and proteinuria are present, indicating a decrease in renal filtration. These are early warning signs of decreased compliance of the heart. The potassium level is normal and the serum creatinine level is normal.

A home health nurse evaluates a client who has chronic obstructive pulmonary disease. Which assessments would the nurse include in this client's evaluation? (Select all that apply.) a. Examination of mucous membranes and nail beds b. Measurement of rate, depth, and rhythm of respirations c. Auscultation of bowel sounds for abnormal sounds d. Check peripheral veins for distention while at rest e. Determine the client's need and use of oxygen f. Ability to perform activities of daily living

ANS: A, B, E, F A home health nurse would assess the client's respiratory status and adequacy of ventilation including an examination of mucous membranes and nail beds for evidence of hypoxia, measurement of rate, depth and rhythm of respirations, auscultation of lung fields for abnormal breath sounds, checking neck veins for distention with the client in a sitting position, and determining the client's needs and use of supplemental oxygen. The home health nurse would also determine the client's ability to perform his or her own ADLs. Auscultation of bowel sounds and assessment of peripheral veins are not part of a focused assessment for a client with COPD.

A client is taking ethambutol for tuberculosis. What instructions does the nurse provide the client regarding this drug? (Select all that apply.) a. Contact the primary health care provider if preexisting gout becomes worse. b. Report any changes in vision immediately to the health care provider. c. Avoid drinking alcoholic beverages due to the chance of liver damage. d. Do not take antacids or eat within 2 hours after taking this medication. e. You will take this medication along with some others for 8 weeks. f. Take this medicine with a full glass of water.

ANS: A, B, E, F The nurse would teach the client that preexisting gout may get worse and the client should report this as medications for gout may need to be adjusted. The nurse would also inform the client about the multi-drug routine used for TB. Optic neuritis can occur with this drug so the client needs to report visual changes right away. The medication should be taken with a full glass of water. Drinking while taking ethambutol causes severe nausea and vomiting. Avoiding antacids and food (within 2 hours) is a precaution with isoniazid.

A nurse collaborates with assistive personnel (AP) to provide care for a client with congestive heart failure. Which instructions would the nurse provide to the AP when delegating care for this client? (Select all that apply.) a. "Reposition the client every 2 hours." b. "Teach the client to perform deep-breathing exercises." c. "Accurately record intake and output." d. "Use the same scale to weigh the client each morning." e. "Place the client on oxygen if the client becomes short of breath."

ANS: A, C, D The AP should reposition the client every 2 hours to improve oxygenation and prevent atelectasis. The AP can also accurately record intake and output, and use the same scale to weigh the client each morning before breakfast. APs are not qualified to teach clients or assess the need for and provide oxygen therapy.

A client is being discharged on warfarin therapy. What discharge instruction is the nurse required to provide? (Select all that apply.) a. Dietary restrictions b. Driving restrictions c. Follow-up laboratory monitoring d. Possible drug-drug interactions e. Reason to take medication f. Wearing a Medic Alert bracelet

ANS: A, C, D, E Best practices state that clients being discharged on warfarin need instruction on follow-up monitoring, dietary restrictions, drug-drug interactions, using a Medic Alert bracelet or necklace, and reason for compliance. Driving is typically not restricted.

A nurse is providing pneumonia vaccinations in a community setting. Due to limited finances, the event organizers must limit giving the vaccination to priority groups. What clients would be considered a priority when administering the pneumonia vaccination? (Select all that apply.) a. A 22-year-old client with asthma b. Client who had a cholecystectomy last year c. Client with well-controlled diabetes d. Healthy 72-year-old client e. Client who is taking medication for hypertension

ANS: A, C, D, E Clients over 65 years of age and any client (no matter what age) with a chronic health condition would be considered a priority for a pneumonia vaccination. Having a cholecystectomy a year ago does not qualify as a chronic health condition.

1A nurse is studying hemodynamic monitoring. Which measurements are correctly matched with the physiologic cause? (Select all that apply.) a. Right atrial pressure 12 mm Hg: right ventricular failure b. Right atrial pressure 4 mm Hg: hypovolemia c. Pulmonary artery pressure 20/10 mm Hg: normal finding d. Pulmonary artery occlusion pressure 20 mm Hg: mitral regurgitation e. Pulmonary artery occlusion pressure 2 mm Hg: afterload reduction

ANS: A, C, D, E Normal right atrial pressure is 0 to 8 mm Hg; high readings can indicate right ventricular failure; low readings often signify hypovolemia. Normal pulmonary artery pressure ranges from 15 to 30 mm Hg systolic to 3 to 12 mm Hg diastolic. Pulmonary artery occlusion pressure ranges from 5 to 12 mm Hg; high values may indicate left ventricular failure, hypervolemia, mitral regurgitation, or intracardiac shunting. A decreased PAOP is seen with hypovolemia or afterload reduction.

A nurse is assessing clients on a rehabilitation unit. Which clients are at greatest risk for airway loss related to aspirated oral and nasopharyngeal secretions? (Select all that apply.) a. A 24 year old with a traumatic brain injury b. A 36 year old who fractured his left femur c. A 58 year old getting radiation therapy d. A 66 year old who is a quadriplegic e. An 80-year-old who is aphasic

ANS: A, C, D, E Thickly crusted, dry secretions that potentially can cause asphyxiation and airway obstruction (inspissated secretions or mucoid impaction) are seen most often in clients who have an altered mental status and level of consciousness (brain injury), are dehydrated, are unable to communicate (aphasic), are unable to cough effectively (quadriplegic), or are at risk for aspiration. The clients with the femur fracture and receiving radiation therapy are not as high of a risk. The location of the radiation is not known.

A nurse reviews a client's laboratory results. Which findings would alert the nurse to the possibility of atherosclerosis? (Select all that apply.) a. Total cholesterol: 280 mg/dL (7.3 mmol/L) b. High-density lipoprotein cholesterol: 50 mg/dL (1.3 mmol/L) c. Triglycerides: 200 mg/dL (2.3 mmol/L) d. Serum albumin: 4 g/dL (5.8 mcmol/L) e. Low-density lipoprotein cholesterol: 160 mg/dL (4.1 mmol/L)

ANS: A, C, E A lipid panel is often used to screen for cardiovascular risk. Total cholesterol, triglycerides, and low-density lipoprotein cholesterol levels are all high, indicating higher risk for cardiovascular disease. High-density lipoprotein cholesterol is within the normal range for both males and females. Serum albumin is not assessed for atherosclerosis.

A nurse cares for a client who is recovering from a right-sided heart catheterization. For which complications of this procedure would the nurse assess? (Select all that apply.) a. Thrombophlebitis b. Stroke c. Pulmonary embolism d. Myocardial infarction e. Cardiac tamponade f. Dysrhythmias

ANS: A, C, E Complications from a right-sided heart catheterization include thrombophlebitis, pulmonary embolism, and vagal response. Cardiac tamponade is a risk of both right- and left-sided heart catheterizations. Stroke, myocardial infarction, and dysrhythmias are complications of left-sided heart catheterizations.

A nurse cares for a client with congestive heart failure who has a regular cardiac rhythm of 128 beats/min. For which physiologic alterations would the nurse assess? (Select all that apply.) a. Decrease in cardiac output b. Increase in cardiac output c. Decrease in blood pressure d. Increase in blood pressure e. Decrease in urine output f. Increase in urine output

ANS: A, C, E Elevated heart rates in a healthy client initially cause blood pressure and cardiac output to increase. However, in a client who has congestive heart failure or a client with long-term tachycardia, ventricular filling time, cardiac output, and blood pressure eventually decrease. As cardiac output and blood pressure decrease, urine output will fall.

A nurse cares for a client who is prescribed an intravenous prostacyclin agent for pulmonary artery hypertension. What actions would the nurse take to ensure the client's safety while on this medication? (Select all that apply.) a. Keep an intravenous line dedicated strictly to the infusion. b. Teach the client that this medication increases pulmonary pressures. c. Ensure that there is always a backup drug cassette available. d. Start a large-bore peripheral intravenous line. e. Use strict aseptic technique when using the drug delivery system.

ANS: A, C, E Intravenous prostacyclin agents would be administered to a client with pulmonary artery hypertension through a central venous catheter with a dedicated intravenous line for this medication. Death has been reported when the drug delivery system is interrupted even briefly; therefore, a backup drug cassette would also be available. The nurse would use strict aseptic technique when using the drug delivery system. The nurse would teach the client that this medication decreases pulmonary pressures and increases lung blood flow.

A nurse is caring for a client who was admitted with hypertrophic cardiomyopathy (HCM). What interprofessional care does the nurse anticipate providing? (Select all that apply.) a. Administering beta blockers b. Administering high-dose furosemide c. Preparing for a cardiac catheterization d. Loading the client on digitalis e. Instructing the client to avoid strenuous exercise f. Teaching the client how to use the CardioMEMS™

ANS: A, C, E Management of obstructive HCM includes administering negative inotropic agents such as beta-adrenergic blocking agents (carvedilol) and calcium antagonists (verapamil). Vasodilators, diuretics, nitrates, and cardiac glycosides are contraindicated in patients with obstructive HCM. Strenuous exercise is also prohibited. Echocardiography, radionuclide imaging, and angiocardiography during cardiac catheterization are performed to diagnose and differentiate cardiomyopathies. The CardioMEMS™ device is used with clients who have heart failure.

A 100-kg client has developed ARDS and needs mechanical ventilation. Which of the following are potentially correct ventilator management choices? (Select all that apply.) a. Tidal volume: 600 mL b. Volume-controlled ventilation c. PEEP based on oxygen saturation d. Suctioning every hour e. High-frequency oscillatory ventilation f. Limited turning for ventilator pressures

ANS: A, C, E The client with ARDS who needs mechanical ventilation benefits from "open lung" and lung protective strategies, such as using low tidal volumes (6 mL/kg body weight). Pressure-controlled ventilation is preferred due to the high pressures often required in these clients. PEEP usually starts at 5 cm H2O and adjusted to keep oxygen saturations in an acceptable range. Suctioning may need to be frequent due to secretions, but is not scheduled hourly. High-frequency oscillatory ventilation is an alternative to traditional modes of ventilation. Early mobility is encouraged as is turning and positioning the client.

A nurse is caring for a client who had coronary artery bypass grafting 2 days ago. What actions does the nurse delegate to the assistive personnel (AP)? (Select all that apply.) a. Assist the client to the chair for meals and to the commode. b. Encourage the client to use the spirometer every 4 hours. c. Ensure that the client wears TED hose or sequential compression devices. d. Have the client rate pain on a 0-10 scale and report to the nurse. e. Take and record a full set of vital signs per hospital protocol.

ANS: A, C, E The nurse can delegate assisting the client to get up in the chair or commode (if the nurse has evaluated the client as being stable), applying TEDs or sequential compression devices, and taking/recording vital signs. The spirometer would be used every hour the day after surgery. Assessing pain using a 0-10 scale is a nursing assessment, although if the client reports pain, the AP would inform the nurse so a more detailed assessment is done.

A nurse assesses a client who has facial trauma. Which assessment findings require immediate intervention? (Select all that apply.) a. Stridor b. Nasal stuffiness c. Edema of the cheek d. Ecchymosis behind the ear e. Eye pain f. Swollen chin

ANS: A, D Stridor is a sign of airway obstruction and requires immediate intervention. Ecchymosis, or bruising, behind the ear is called "battle sign" and indicates basilar skull fracture. Nasal stuffiness, edema of the cheek or chin, and eye pain do not interfere with respirations or neurologic function, and therefore are not priorities for immediate intervention.

A nurse is caring for a client who has a tracheostomy tube. What actions may the nurse delegate to assistive personnel (AP)? (Select all that apply.) a. Applying water-soluble lip balm to the client's lips b. Ensuring that the humidification provided is adequate c. Performing oral care with alcohol-based mouthwash d. Reminding the client to cough and deep breathe often e. Suctioning excess secretions through the tracheostomy f. Holding the new tracheostomy tube while the RN changes the ties

ANS: A, D The AP can perform hygiene measures such as applying lip balm and reinforce teaching such as reminding the client to perform coughing and deep-breathing exercises. Oral care can be accomplished with normal saline, not products that dry the mouth. Ensuring that the humidity is adequate and suctioning through the tracheostomy are nursing functions. When needed, a second licensed person assists with holding the tracheostomy tube during tie changes; some hospitals require a second licensed person during the first 72 hours after placement.

A nurse assesses a client who is prescribed varenicline for smoking cessation. Which signs or symptoms would the nurse identify as adverse effects of this medication? (Select all that apply.) a. Visual hallucinations b. Tachycardia c. Decreased cravings d. Manic behavior e. Increased thirst f. Orangish urine

ANS: A, D Varenicline has a black box warning stating that the drug can cause manic behavior and hallucinations. The nurse would assess for changes in behavior and thought processes, including manic behaviors and visual hallucinations. Tachycardia, increased thirst, and orange-colored urine are not adverse effects of this medication. Decreased cravings are a therapeutic response to this medication.

After teaching a client with congestive heart failure (CHF), the nurse assesses the client's understanding. Which client statements indicate a correct understanding of the teaching related to nutritional intake? (Select all that apply.) a. "I'll read the nutritional labels on food items for salt content." b. "I will drink at least 3 L of water each day." c. "Using salt in moderation will reduce the workload of my heart." d. "I will eat oatmeal for breakfast instead of ham and eggs." e. "Substituting fresh vegetables for canned ones will lower my salt intake." f. "Salt substitutes are a good way to cut down on sodium in my diet."

ANS: A, D, E Nutritional therapy for a client with CHF is focused on decreasing sodium and water retention to decrease the workload of the heart. The client would be taught to read nutritional labels on all food items, omit table salt and foods high in sodium (e.g., ham and canned foods), and limit water intake to a normal 2 L/day. Salt substitutes typically contain potassium, so although they are not strictly banned, clients would have to have their renal function and serum potassium monitored while using them. It would be safer to avoid them.

A client is being discharged home after having a tracheostomy placed. What suggestions does the nurse offer to help the client maintain self-esteem? (Select all that apply.) a. Create a communication system. b. Don't go out in public alone. c. Find hobbies to enjoy at home. d. Try loose-fitting shirts with collars. e. Wear fashionable scarves.

ANS: A, D, E The client with a tracheostomy may be shy and hesitant to go out in public. The client needs to have a sound communication method to ease frustration. The nurse can also suggest ways of enhancing appearance so the client is willing to leave the house. These can include wearing scarves and loose-fitting shirts to hide the stoma. Keeping the client homebound is not good advice.

A nurse collaborates with a respiratory therapist to complete pulmonary function tests (PFTs) for a client. Which statements would the nurse include in communications with the respiratory therapist prior to the tests? (Select all that apply.) a. "I held the client's morning bronchodilator medication." b. "The client is ready to go down to radiology for this examination." c. "Physical therapy states the client can run on a treadmill." d. "I advised the client not to smoke for 6 hours prior to the test." e. "The client is alert and can follow your commands."

ANS: A, D, E To ensure that the PFTs are accurate, the therapist needs to know that no bronchodilators have been administered in the past 4 to 6 hours (depending on the suspected cause), the client did not smoke within 6 to 8 hours prior to the test, and the client can follow basic commands, including different breathing maneuvers. The respiratory therapist can perform PFTs at the bedside or the respiratory lab. A treadmill is not used for this test.

The nurse working in the emergency department knows that which factors are commonly related to aneurysm formation? (Select all that apply.) a. Atherosclerosis b. Down syndrome c. Frequent heartburn d. History of hypertension e. History of smoking f. Hyperlipidemia

ANS: A, D, E, F Atherosclerosis, hypertension, hyperlipidemia, hyperlipidemia, and smoking are the most commonly related factors. Down syndrome and heartburn have no relation to aneurysm formation.

A nurse teaches a client who is interested in smoking cessation. Which statements would the nurse include in this client's teaching? (Select all that apply.) a. "Find an activity that you enjoy and will keep your hands busy." b. "Keep snacks like potato chips on hand to nibble on." c. "Identify a consequence for yourself in case you backslide." d. "Drink at least eight glasses of water each day." e. "Make a list of reasons you want to stop smoking." f. "Set a quit date and stick to it."

ANS: A, D, E, F The nurse would teach a client who is interested in smoking cessation to find an activity that keeps the hands busy, to keep healthy snacks on hand to nibble on, to drink at least eight glasses of water each day, to make a list of reasons for quitting smoking, and to set a firm quit date and stick to it. The nurse would also encourage the client not to be upset if he or she backslides and has a cigarette but to try to determine what conditions caused him or her to smoke.

A nurse teaches a client with heart failure about energy conservation. Which statement would the nurse include in this client's teaching? a. "Walk until you become short of breath, and then walk back home." b. "Begin walking 200 feet a day three times a week." c. "Do not lift heavy weights for 6 months." d. "Eat plenty of protein to build your strength."

ANS: B A client who has heart failure would be taught to conserve energy and given an exercise plan. The client should begin walking 200-400 feet a day at home three times a week. The client should not walk until becoming short of breath because he or she may not make it back home. The lifting restriction is specifically for clients after valve replacements. Protein does help build strength, but this direction is not specific to heart failure.

A nurse is caring for a client with a deep vein thrombosis (DVT). What nursing assessment indicates that an important outcome has been met? a. Ambulates with assistance b. Oxygen saturation of 98% c. Pain of 2/10 after medication d. Verbalizing risk factors

ANS: B A critical complication of DVT is pulmonary embolism. A normal oxygen saturation indicates that this has not occurred. The other assessments are also positive, but not as important.

A client has intra-arterial blood pressure monitoring after a myocardial infarction. The nurse notes that the client's heart rate has increased from 88 to 110 beats/min, and the blood pressure dropped from 120/82 to 100/60 mm Hg. What action by the nurse is most appropriate? a. Allow the client to rest quietly. b. Assess the client for bleeding. c. Document the findings in the chart. d. Medicate the client for pain.

ANS: B A major complication related to intra-arterial blood pressure monitoring is hemorrhage from the insertion site. Since these vital signs are out of the normal range, are a change, and are consistent with blood loss, the nurse would assess the client for any bleeding associated with the arterial line. The nurse would document the findings after a full assessment. The client may or may not need pain medication and rest; the nurse first needs to rule out any emergent bleeding.

A nurse is assessing a client who is recovering from a lung biopsy. The client's breath sounds are absent. While another nurse calls the Rapid Response Team, what action by the nurse takes is most important? a. Take a full set of vital signs. b. Obtain pulse oximetry reading. c. Ask the patient about hemoptysis. d. Inspect the biopsy site.

ANS: B Absent breath sounds may indicate that the client has a pneumothorax, a serious complication after a needle biopsy or open lung biopsy. The nurse would first obtain a pulse oximetry reading and perform other respiratory assessments. Temperature is not a priority. The nurse can ask about other symptoms while conducting the assessment. The nurse would assess the biopsy site and/or dressings, but this is not the first action.

A client received tissue plasminogen activator (tPA) after a myocardial infarction and now is on an intravenous infusion of heparin. The client's spouse asks why the client needs this medication. What response by the nurse is best? a. "The t-PA didn't dissolve the entire coronary clot." b. "The heparin keeps that artery from getting blocked again." c. "Heparin keeps the blood as thin as possible for a longer time." d. "The heparin prevents a stroke from occurring as the t-PA wears off."

ANS: B After the original intracoronary clot has dissolved, large amounts of thrombin are released into the bloodstream, increasing the chance of the vessel reoccluding. The other statements are not accurate. Heparin is not a "blood thinner," although laypeople may refer to it as such.

A client has hypertension and high risk factors for cardiovascular disease. The client is overwhelmed with the recommended lifestyle changes. What action by the nurse is best? a. Assess the client's support system. b. Assist in finding one change the client can control. c. Determine what stressors the client faces in daily life. d. Inquire about delegating some of the client's obligations.

ANS: B All options are appropriate when assessing stress and responses to stress. However, this client feels overwhelmed by the suggested lifestyle changes. Instead of looking at all the needed changes, the nurse would assist the client in choosing one the client feels optimistic about controlling. Once the client has mastered that change, he or she can move forward with another change. Determining support systems, daily stressors, and delegation opportunities does not directly impact the client's feelings of control.

20. After assessing a client who is receiving an amiodarone intravenous infusion for unstable ventricular tachycardia, the nurse documents the findings and compares these with the previous assessment findings: Vital Signs Time: 08:00 Temperature: 98° F (36.7° C) Heart rate: 68 beats/min Blood pressure: 135/60 mm Hg Respiratory rate: 14 breaths/min Oxygen saturation: 96% Oxygen therapy: 2 L nasal cannula Time: 10:00 Temperature: 98.2° F (36.8° C) Heart rate: 50 beats/min Blood pressure: 132/57 mm Hg Respiratory rate: 16 breaths/min Oxygen saturation: 95% Oxygen therapy: 2 L nasal cannula Nursing Assessment Time: 08:00 Client alert and oriented. Cardiac rhythm: normal sinus rhythm. Skin: warm, dry, and appropriate for race. Respirations equal and unlabored. Client denies shortness of breath and chest pain. Time: 10:00 Client alert and oriented. Cardiac rhythm: sinus bradycardia. Skin: warm, dry, and appropriate for race. Respirations equal and unlabored. Client denies shortness of breath and chest pain. Client voids 420 mL of clear yellow urine. Based on the assessments, what action would the nurse take? a. Stop the infusion and flush the IV. b. Slow the amiodarone infusion rate. c. Administer IV normal saline. d. Ask the client to cough and deep breathe.

ANS: B Amiodarone lengthens the absolute refractory period and prolongs repolarization and the action potential duration (and heart rate), so IV administration of amiodarone may cause bradycardia and atrioventricular (AV) block. The correct action for the nurse to take at this time is to slow the infusion, because the client is asymptomatic and no evidence reveals AV block that might require pacing. Abruptly ceasing the medication could allow fatal dysrhythmias to occur. The administration of IV fluids and encouragement of coughing and deep-breathing exercises are not indicated, and will not increase the client's heart rate.

A nurse wants to provide community service that helps meet the goals of Healthy People 2020 (HP2020) related to cardiovascular disease and stroke. What activity would best meet this goal? a. Teach high school students heart-healthy living. b. Participate in blood pressure screenings at the mall. c. Provide pamphlets on heart disease at the grocery store. d. Set up an "Ask the nurse" booth at the pet store.

ANS: B An important goal of HP2020 is to increase the proportion of adults who have had their blood pressure measured within the preceding 2 years and can state whether their blood pressure was normal or high. Participating in blood pressure screening in a public spot will best help meet that goal. The other options are all appropriate but do not specifically help meet a goal.

A nurse is caring for four clients. Which one would the nurse see first? a. Client who needs a beta blocker, and has a blood pressure of 98/58 mm Hg. b. Client who had a first dose of captopril and needs to use the bathroom. c. Hypertensive client with a blood pressure of 188/92 mm Hg. d. Client who needs pain medication prior to a dressing change of a surgical wound.

ANS: B Angiotensin-converting enzyme inhibitors such as captopril can cause hypotension, especially after the first dose. The nurse would see this client first to prevent falling if the client decides to get up without assistance. The two blood pressure readings are abnormal but not critical. The nurse would check on the client with higher blood pressure next to assess for problems related to the reading. The nurse can administer the beta blocker as standards state to hold it if the systolic blood pressure is below 90 to 100 mm Hg. The client who needs pain medication prior to the dressing change is not a priority over client safety and assisting the other client to the bathroom.

A nurse cares for a client with arthritis who reports frequent asthma attacks. What action would the nurse take first? a. Review the client's pulmonary function test results. b. Ask about medications the client is currently taking. c. Assess how frequently the client uses a bronchodilator. d. Consult the primary health care provider and request arterial blood gases.

ANS: B Aspirin and other nonsteroidal anti-inflammatory drugs (NSAIDs) can trigger asthma in some people. This results from increased production of leukotriene when aspirin or NSAIDs suppress other inflammatory pathways and is a likely culprit given the client's history. Reviewing pulmonary function test results will not address the immediate problem of frequent asthma attacks. This is a good time to review response to bronchodilators, but assessing triggers is more important. Questioning the client about the use of bronchodilators will address interventions for the attacks but not their cause. Reviewing arterial blood gas results would not be of use in a client between attacks because many clients are asymptomatic when not having attacks.

A nurse assesses a client who has mitral valve regurgitation. For which cardiac dysrhythmia would the nurse assess? a. Preventricular contractions b. Atrial fibrillation c. Symptomatic bradycardia d. Sinus tachycardia

ANS: B Atrial fibrillation is a clinical manifestation of mitral valve regurgitation and stenosis. Preventricular contractions and bradycardia are not associated with valvular problems. These are usually identified in clients with electrolyte imbalances, myocardial infarction, and sinus node problems. Sinus tachycardia is a manifestation of aortic regurgitation due to a decrease in cardiac output.

A nurse is assessing clients on a medical-surgical unit. Which client would the nurse identify as being at greatest risk for atrial fibrillation? a. A 45-year-old who takes an aspirin daily. b. A 50-year-old who is post coronary artery bypass graft surgery. c. A 78-year-old who had a carotid endarterectomy. d. An 80-year-old with chronic obstructive pulmonary disease.

ANS: B Atrial fibrillation occurs commonly in clients with cardiac disease. Other risk factors include hypertension (HTN), previous ischemic stroke, transient ischemic attack (TIA) or other thromboembolic event, diabetes mellitus, heart failure, obesity, hyperthyroidism, chronic kidney disease, excessive alcohol use, and mitral valve disease. The other conditions do not place these clients at higher risk for atrial fibrillation.

A nurse evaluates prescriptions for a client with chronic atrial fibrillation. Which medication would the nurse expect to find on this client's medication administration record to prevent a common complication of this condition? a. Sotalol b. Warfarin c. Atropine d. Lidocaine

ANS: B Atrial fibrillation puts clients at risk for developing emboli. Clients at risk for emboli are treated with anticoagulants, such as heparin, enoxaparin, or warfarin. Sotalol, atropine, and lidocaine are not appropriate for preventing this complication.

A nurse cares for a client who has a heart rate averaging 56 beats/min with no adverse symptoms. Which activity modification would the nurse suggest to avoid further slowing of the heart rate? a. "Make certain that your bath water is warm." b. "Avoid straining while having a bowel movement." c. "Limit your intake of caffeinated drinks to one a day." d. "Avoid strenuous exercise such as running."

ANS: B Bearing down strenuously during a bowel movement is one type of Valsalva maneuver, which stimulates the vagus nerve and results in slowing of the heart rate. Such a response is not desirable in a person who has bradycardia. The other instructions are not appropriate for this condition.

A client has presented to the emergency department with an acute myocardial infarction (MI). What action by the nurse is best for optimal client outcomes? a. Obtain an electrocardiogram (ECG) within 20 minutes. b. Give the client a nonenteric coated aspirin. c. Notify the Rapid Response Team immediately. d. Prepare to administer thrombolytics within 30 minutes.

ANS: B Best practice recommendations for acute MI require that aspirin is administered when a client with MI presents to the emergency department or when an MI occurs in the hospital. A rapid ECG (within 10 minutes) is vital for best outcomes. The Rapid Response Team is not needed if an emergency department provider is available. Thrombolytics may or may not be needed depending on the type of myocardial infarction the client has.

A nurse administers medications to a client who has asthma. Which medication classification is paired correctly with its physiologic action? a. Bronchodilator—stabilizes the membranes of mast cells and prevents the release of inflammatory mediators. b. Cholinergic antagonist—causes bronchodilation by inhibiting the parasympathetic nervous system. c. Corticosteroid—relaxes bronchiolar smooth muscles by binding to and activating pulmonary beta2 receptors. d. Cromone—disrupts the production of pathways of inflammatory mediators.

ANS: B Cholinergic antagonist drugs cause bronchodilation by inhibiting the parasympathetic nervous system. This allows the sympathetic nervous system to dominate and release norepinephrine that activates beta2receptors. Bronchodilators relax bronchiolar smooth muscles by binding to and activating pulmonary beta2 receptors. Corticosteroids disrupt the production of pathways of inflammatory mediators. Cromones stabilize the membranes of mast cells and prevent the release of inflammatory mediators.

A nurse teaches a client with diabetes mellitus and a body mass index of 42 who is at high risk for coronary artery disease. Which statement related to nutrition would the nurse include in this client's teaching? a. "The best way to lose weight is a high-protein, low-carbohydrate diet." b. "You should balance weight loss with consuming necessary nutrients." c. "A nutritionist will provide you with information about your new diet." d. "If you exercise more frequently, you won't need to change your diet."

ANS: B Clients at risk for cardiovascular diseases should follow the American Heart Association guidelines to combat obesity and improve cardiac health. The nurse would encourage the client to eat vegetables, fruits, unrefined whole-grain products, and fat-free dairy products while losing weight. High-protein food items are often high in fat and calories. Although the nutritionist can assist with client education, the nurse would include nutrition education and assist the client to make healthy decisions. Exercising and eating nutrient-rich foods are both important components in reducing cardiovascular risk.

A client is receiving an infusion of alteplase for an intra-arterial clot. The client begins to mumble and is disoriented. What action by the nurse is most important? a. Assess the client's neurologic status. b. Notify the Rapid Response Team. c. Prepare to administer vitamin K. d. Turn down the infusion rate.

ANS: B Clients on fibrinolytic therapy are at high risk of bleeding. The sudden onset of neurologic signs may indicate that the client is having a hemorrhagic stroke. The nurse does need to complete a thorough neurologic examination, but would first call the Rapid Response Team based on the client's manifestations. Vitamin K is not the antidote for this drug. Turning down the infusion rate will not be helpful if the client is still receiving any of the drug.

After teaching a client who is being discharged home after mitral valve replacement surgery, the nurse assesses the client's understanding. Which client statement indicates a need for additional teaching? a. "I'll be able to carry heavy loads after 6 months of rest." b. "I will have my teeth cleaned by my dentist in 2 weeks." c. "I must avoid eating foods high in vitamin K, like spinach." d. "I must use an electric razor instead of a straight razor to shave."

ANS: B Clients who have defective or repaired valves are at high risk for endocarditis. The client who has had valve surgery should avoid dental procedures for 6 months because of the risk for endocarditis. When undergoing a mitral valve replacement surgery, the client needs to be placed on anticoagulant therapy to prevent vegetation forming on the new valve. Clients on anticoagulant therapy would be instructed on bleeding precautions, including using an electric razor. If the client is prescribed warfarin, the client should avoid foods high in vitamin K. Clients recovering from open-heart valve replacements should not carry anything heavy for 6 months while the chest incision and muscle heal.

A nurse is providing discharge teaching to a client recovering from a heart transplant. Which statement would the nurse include? a. "Use a soft-bristled toothbrush and avoid flossing." b. "Avoid large crowds and people who are sick." c. "Change positions slowly to avoid hypotension." d. "Check your heart rate before taking the medication."

ANS: B Clients who have had heart transplants must take immunosuppressant therapy for the rest of their lives. The nurse would teach this client to avoid crowds and sick people to reduce the risk of becoming ill him- or herself. These medications do not place clients at risk for bleeding, orthostatic hypotension, or changes in heart rate. Orthostatic hypotension from the denervated heart is generally only a problem in the immediate postoperative period.

A nurse assesses a client who has a history of heart failure. Which question would the nurse ask to assess the extent of the client's heart failure? a. "Do you have trouble breathing or chest pain?" b. "Are you still able to walk upstairs without fatigue?" c. "Do you awake with breathlessness during the night?" d. "Do you have new-onset heaviness in your legs?"

ANS: B Clients with a history of heart failure generally have negative findings, such as shortness of breath and fatigue. The nurse needs to determine whether the client's activity is the same or worse, or whether the client identifies a decrease in activity level. Trouble breathing, chest pain, breathlessness at night, and peripheral edema are symptoms of heart failure, but do not provide data that can determine the extent of the client's heart failure.

A nurse assesses a client with atrial fibrillation. Which manifestation would alert the nurse to the possibility of a serious complication from this condition? a. Sinus tachycardia b. Speech alterations c. Fatigue d. Dyspnea with activity

ANS: B Clients with atrial fibrillation are at risk for embolic stroke. Evidence of embolic events includes changes in mentation, speech, sensory function, and motor function. Clients with atrial fibrillation often have a rapid ventricular response as a result. Fatigue is a nonspecific complaint. Clients with atrial fibrillation often have dyspnea as a result of the decreased cardiac output caused by the rhythm disturbance.

A client has a tracheostomy tube in place. When the nurse suctions the client, food particles are noted. What action by the nurse is best? a. Elevate the head of the client's bed. b. Measure and compare cuff pressures. c. Place the client on NPO status. d. Request that the client have a swallow study.

ANS: B Constant pressure from the tracheostomy tube cuff can cause tracheomalacia, leading to dilation of the tracheal passage. This can be manifested by food particles seen in secretions or by noting that larger and larger amounts of pressure are needed to keep the tracheostomy cuff inflated. The nurse would measure the pressures and compare them to previous ones to detect a trend. Elevating the head of the bed, placing the client on NPO status, and requesting a swallow study will not correct this situation.

A nurse is caring for a client who received benzocaine spray prior to a recent bronchoscopy. The client presents with continuous cyanosis even with oxygen therapy. What action would the nurse take next? a. Administer an albuterol treatment. b. Notify the Rapid Response Team. c. Assess the client's peripheral pulses. d. Obtain blood and sputum cultures.

ANS: B Cyanosis unresponsive to oxygen therapy is a sign of methemoglobinemia, which is an adverse effect of benzocaine spray. This condition can lead to death. The nurse would notify the Rapid Response Team to provide advanced care. An albuterol treatment would not address the client's oxygenation problem. Assessment of pulses and cultures will not provide data necessary to treat the client.

A clinic nurse is reviewing care measures with a client who has asthma, Step 3. What statement by the client indicates the need to review the information? a. "I still will use my rapid-acting inhaler for an asthma attack." b. "I will always use the spacer with my dry powder inhaler." c. "If I am stable for 3 months, I might be able to reduce my drugs." d. "My inhaled corticosteroid must be taken regularly to work well."

ANS: B Dry powder inhalers are not used with a spacer. The other statements are accurate.

A nurse assesses a client with mitral valve stenosis. What clinical sign or symptom would alert the nurse to the possibility that the client's stenosis has progressed? a. Oxygen saturation of 92% b. Dyspnea on exertion c. Muted systolic murmur d. Upper extremity weakness

ANS: B Dyspnea on exertion develops as the mitral valvular orifice narrows and pressure in the lungs increases. The other signs and symptoms do not relate to the progression of mitral valve stenosis.

A client is on intravenous heparin to treat a pulmonary embolism. The client's most recent partial thromboplastin time (PTT) was 25 seconds. What order would the nurse anticipate? a. Decrease the heparin rate. b. Increase the heparin rate. c. No change to the heparin rate. d. Stop heparin; start warfarin.

ANS: B For clients on heparin, a PTT of 1.5 to 2.5 times the normal value is needed to demonstrate that the heparin is working. A normal PTT is 25 to 35 seconds, so this client's PTT value is too low. The heparin rate needs to be increased. Warfarin is not indicated in this situation.

A nurse assists with the cardioversion of a client experiencing acute atrial fibrillation. What action would the nurse take prior to the cardioversion? a. Administer intravenous adenosine. b. Turn off oxygen therapy. c. Ensure that a tongue blade is available. d. Position the client on the left side.

ANS: B For safety during cardioversion, the nurse would turn off any oxygen therapy to prevent fire. The other interventions are not appropriate for a cardioversion. The client would be placed in a supine position.

A nurse is preparing to admit a client on mechanical ventilation for acute respiratory failure from the emergency department. What action does the nurse take first? a. Assessing that the ventilator settings are correct b. Ensuring that there is a bag-valve-mask in the room c. Obtaining personal protective equipment d. Planning to suction the client upon arrival to the room

ANS: B Having a bag-valve-mask device is critical in case the client needs manual breathing. The respiratory therapist is usually primarily responsible for setting up the ventilator, although the nurse would know and check the settings. Personal protective equipment is important, but ensuring client safety is the most important action. The client may or may not need suctioning on arrival.

A nurse is in charge of the coronary intensive care unit. Which client would the nurse see first? a. Client on a nitroglycerin infusion at 5 mcg/min, not titrated in the last 4 hours b. Client who is 1-day post coronary artery bypass graft, with blood pressure 88/64 mm Hg c. Client who is 1-day post percutaneous coronary intervention, going home this morning d. Client who is 2-day post coronary artery bypass graft, who became dizzy this morning while walking

ANS: B Hypotension after coronary artery bypass graft surgery can be dangerous because it can lead to collapse of the graft. The charge nurse would see this client first. The client who became dizzy earlier would be seen next. The client on the nitroglycerin drip is stable. The client going home can wait until the other clients are cared for.

After administering the first dose of captopril to a client with heart failure, the nurse implements interventions to decrease complications. Which intervention is most important for the nurse to implement? a. Provide food to decrease nausea and aid in absorption. b. Instruct the client to ask for assistance when rising from bed. c. Collaborate with assistive personnel to bathe the client. d. Monitor potassium levels and check for symptoms of hypokalemia.

ANS: B Hypotension is a side effect of ACE inhibitors such as captopril. Clients with a fluid volume deficit should have their volume replaced or start at a lower dose of the drug to minimize this effect. The nurse would instruct the client to seek assistance before arising from bed to prevent injury from postural hypotension. ACE inhibitors do not need to be taken with food. Collaboration with assistive personnel to provide hygiene is not a priority. The client would be encouraged to complete activities of daily living as independently as possible. The nurse would monitor for hyperkalemia, not hypokalemia, especially if the client has renal insufficiency secondary to heart failure.

A client has been taking isoniazid for tuberculosis for 3 weeks. What laboratory results need to be reported to the primary health care provider immediately? a. Albumin: 5.1 g/dL (7.4 mcmol/L) b. Alanine aminotransferase (ALT): 180 U/L c. Red blood cell (RBC) count: 5.2/million/µL (5.2 1012/L) d. White blood cell (WBC) count: 12,500/mm3 (12.5 109/L)

ANS: B INH can cause liver damage, especially if the client drinks alcohol. The ALT (one of the liver enzymes) is extremely high and needs to be reported immediately. The albumin and RBCs are normal. The WBCs are slightly high, but that would be an expected finding in a client with an infection.

A client has hemodynamic monitoring after a myocardial infarction. What safety precaution does the nurse implement for this client? a. Document pulmonary artery occlusion pressure (PAOP) readings and assess their trends. b. Ensure that the balloon does not remain wedged. c. Keep the client on strict NPO status. d. Maintain the client in a semi-Fowler position.

ANS: B If the balloon remains inflated, it can cause pulmonary infarction or rupture. The nurse would ensure that the balloon remains deflated between PAOP readings. Documenting PAOP readings and assessing trends are important nursing actions related to hemodynamic monitoring, but are not specifically related to safety. The client does not have to be NPO while undergoing hemodynamic monitoring. Positioning is not related to safety with hemodynamic monitoring.

The nurse is caring for a client with a chest tube after a coronary artery bypass graft. The drainage stops suddenly. What action by the nurse is most important? a. Increase the setting on the suction. b. Notify the primary health care provider immediately. c. Reposition the chest tube. d. Take the tubing apart to assess for clots.

ANS: B If the drainage in the chest tube decreases significantly and dramatically, the tube may be blocked by a clot. This could lead to cardiac tamponade. The nurse would notify the primary health care provider immediately. The nurse would not independently increase the suction, reposition the chest tube, or take the tubing apart.

While assessing a client who is 12 hours postoperative after a thoracotomy for lung cancer, a nurse notices that the chest tube is dislodged. Which action by the nurse is best? a. Assess for drainage from the site. b. Cover the insertion site with sterile gauze. c. Contact the primary health care provider. d. Reinsert the tube using sterile technique.

ANS: B Immediately covering the insertion site helps prevent air from entering the pleural space and causing a pneumothorax. The area will not reseal quickly enough to prevent air from entering the chest. The nurse would not leave the client to obtain a suture kit. An occlusive dressing may cause a tension pneumothorax. The nurse does not need to assess the site at this moment. The primary health care provider would be called to reinsert the chest tube or prescribe other treatment options.

A nurse cares for a client with an intravenous temporary pacemaker for bradycardia. The nurse observes the presence of a pacing spike but no QRS complex on the client's electrocardiogram. What action would the nurse take next? a. Administer intravenous diltiazem. b. Assess vital signs and level of consciousness. c. Administer sublingual nitroglycerin. d. Assess capillary refill and temperature.

ANS: B In temporary pacing, the wires are threaded onto the epicardial surface of the heart and exit through the chest wall. The pacemaker spike would be followed immediately by a QRS complex. Pacing spikes seen without subsequent QRS complexes imply loss of capture. If there is no capture, then there is no ventricular depolarization and contraction. The nurse would assess for cardiac output via vital signs and level of consciousness. The other interventions would not determine if the client is tolerating the loss of capture.

An older adult is brought to the emergency department by a family member, who reports a moderate change in mental status and mild cough. The client is afebrile. The primary health care provider orders a chest x-ray. The family member questions why this is needed since the symptoms seem so vague. What response by the nurse is best? a. "Chest x-rays are always ordered when we suspect pneumonia." b. "Older people often have vague symptoms, so an x-ray is essential." c. "The x-ray can be done and read before laboratory work is reported." d. "We are testing for any possible source of infection in the client."

ANS: B It is essential to obtain an early chest x-ray in older adults suspected of having pneumonia because symptoms are often vague. Waiting until definitive signs and symptoms are present to obtain the x-ray leads to a costly delay in treatment. Stating that chest x-rays are always ordered does not give the family definitive information. The x-ray can be done while laboratory values are still pending, but this also does not provide specific information about the importance of a chest x-ray in this client. The client has symptoms of pneumonia, so the staff is not testing for any possible source of infection but rather is testing for a suspected disorder.

A nurse cares for a client who had a partial laryngectomy 10 days ago. The client states that all food tastes bland. How would the nurse respond? a. "I will consult the speech therapist to ensure you are swallowing properly." b. "This is normal after surgery. What types of food do you like to eat?" c. "I will ask the dietitian to change the consistency of the food in your diet." d. "Replacement of protein, calories, and water is very important after surgery."

ANS: B Many clients experience changes in taste after surgery. The nurse would identify foods that the client wants to eat to ensure that the client maintains necessary nutrition. Although the nurse would collaborate with the speech therapist and dietitian to ensure appropriate replacement of protein, calories, and water, the other responses do not address the patient's concerns.

A client has been diagnosed with hypertension but does not take the antihypertensive medications because of a lack of symptoms. What response by the nurse is best? a. "Do you have trouble affording your medications?" b. "Most people with hypertension do not have symptoms." c. "You are lucky; most people get severe morning headaches." d. "You need to take your medicine or you will get kidney failure."

ANS: B Most people with hypertension are asymptomatic, although a small percentage do have symptoms such as headache. The nurse would explain this to the client. Asking about paying for medications utilizes closed-ended questioning and is not therapeutic. Threatening the client with possible complications will not increase compliance.

A client is hospitalized with a second episode of pulmonary embolism (PE). Recent genetic testing reveals that the client has an alteration in the gene CYP2C19. What action by the nurse is best? a. Instruct the client to eliminate all vitamin K from the diet. b. Prepare preoperative teaching for an inferior vena cava (IVC) filter. c. Refer the client to a chronic illness support group. d. Teach the client to use a soft-bristled toothbrush.

ANS: B Often clients are discharged from the hospital on warfarin after a PE. However, clients with a variation in the CYP2C19 gene do not metabolize warfarin well and have much higher blood levels and more side effects. This client is a poor candidate for warfarin therapy, and the prescriber will most likely order an IVC filter device to be implanted. The other option is to lower the dose of warfarin. The nurse would prepare to do preoperative teaching on this procedure. It would be impossible to eliminate all vitamin K from the diet. A chronic illness support group may be needed, but this is not the best intervention as it is not as specific to the client as the IVC filter. A soft-bristled toothbrush is a safety measure for clients on anticoagulation therapy.

A client with coronary artery disease (CAD) asks the nurse about taking fish oil supplements. What response by the nurse is best? a. "Fish oil is contraindicated with most drugs for CAD." b. "The best source is fish, but pills have benefits too." c. "There is no evidence to support fish oil use with CAD." d. "You can reverse CAD totally with diet and supplements."

ANS: B Omega-3 fatty acids have shown benefit in reducing lipid levels, in reducing the incidence of sudden cardiac death, and for stabilizing atherosclerotic plaque. The preferred source of omega-3 acids is from fish rich in long-chain n-3 polyunsaturated fatty acids two times a week or a daily fish oil nutritional supplement (1 to 2 g/day). The other options are not accurate.

A client is 4 hours postoperative after a femoral-popliteal bypass. The client reports throbbing leg pain on the affected side, rated as 7/10. What action by the nurse is most important? a. Administer pain medication as ordered. b. Assess distal pulses and skin color. c. Document the findings in the client's chart. d. Notify the surgeon immediately.

ANS: B Once perfusion has been restored or improved to an extremity, clients can often feel a throbbing pain due to the increased blood flow. However, it is important to differentiate this pain from ischemia. The nurse would assess for other signs of perfusion, such as distal pulses and skin color/temperature. Administering pain medication is done once the nurse determines that the client's perfusion status is normal. Documentation needs to be thorough. Notifying the surgeon is not necessary.

A nurse caring for a client removes the client's oxygen as prescribed. The client is now breathing what percentage of oxygen in the room air? a. 14% b. 21% c. 28% d. 31%

ANS: B Oxygen content of atmospheric or "room air" is about 21%.

A client is wearing a Venturi mask to deliver oxygen and the dinner tray has arrived. What action by the nurse is best? a. Assess the client's oxygen saturation and, if normal, turn off the oxygen. b. Determine if the client can switch to a nasal cannula during the meal. c. Have the client lift the mask off the face when taking bites of food. d. Turn the oxygen off while the client eats the meal and then restart it.

ANS: B Oxygen is a drug that needs to be delivered constantly. The nurse would determine if the primary health care provider has approved switching to a nasal cannula during meals. If not, the nurse would consult with the primary health care provider about this issue. The primary health care provider would need to prescribe discontinuing oxygen if the client's oxygen saturation is normal. The oxygen would not be turned off. Lifting the mask to eat will alter the FiO2 delivered.

A nurse is caring for a client using oxygen while in the hospital. What assessment finding indicates that outcomes for client safety with oxygen therapy are being met? a. 100% of meals being eaten by the client b. Intact skin behind the ears c. The client understanding the need for oxygen d. Unchanged weight for the past 3 days

ANS: B Oxygen tubing can cause pressure injuries, so clients using oxygen have a high risk of skin breakdown. Intact skin behind the ears indicates that goals for maintaining client safety with oxygen therapy are being met. Nutrition and weight are not related to using oxygen. Understanding the need for oxygen is important but would not take priority over a physical problem.

The charge nurse on a medical unit is preparing to admit several "clients" who have possible pandemic flu during a preparedness drill. What action by the nurse is best? a. Admit the "clients" on Contact Precautions. b. Inquire as to recent travel outside the United States. c. Do not allow pregnant caregivers to care for these "clients." d. Place the "clients" on enhanced Droplet Precautions.

ANS: B Preventing the spread of pandemic flu is equally important as caring for the clients who have it. Preventing the spread of disease is vital. The nurse would ask the "clients" about recent overseas travel to assess the risk of a pandemic flu. Clients with possible pandemic flu need to be in Contact and Airborne Precautions the infectious organism is identified and routes of transmission known. There is no specific danger to pregnant caregivers. Droplet Precautions are not appropriate.

A nurse teaches a client to use a room humidifier after a laryngectomy. Which statement would the nurse include in this patient's teaching? a. "Add peppermint oil to the humidifier to relax the airway." b. "Make sure you clean the humidifier to prevent infection." c. "Keep the humidifier filled with water at all times." d. "Use the humidifier when you sleep, even during daytime naps."

ANS: B Priority teaching related to the use of a room humidifier focuses on infection control. Clients would be taught to meticulously clean the humidifier to prevent the spread of mold or other sources of infection. Peppermint oil would not be added to a humidifier. The humidifier would be refilled with water as needed and would be used while awake and asleep.

A client with ARDS is receiving minimal amounts of IV fluids. The new nurse notes the client is scheduled to receive a diuretic at this time. The nurse consults the Staff Development Nurse to determine the best course of action. What will the new nurse do? a. Contact the primary health care provider. b. Give the ordered diuretic as scheduled. c. Request an increase in the IV rate. d. Calculate the client's 24-hour fluid balance.

ANS: B Research has shown that clients with ARDS may benefit from conservative fluid therapy along with diuretics to maintain fluid balance. The nurse will give the ordered diuretic as scheduled. There is no reason to contact the provider or request an increased IV rate. The nurse can calculate the 24-hour fluid balance, but this will not influence the administration of the medication.

1. A nurse assesses a client admitted to the cardiac unit. Which statement by the client alerts the nurse to the possibility of right-sided heart failure? a. "I sleep with four pillows at night." b. "My shoes fit really tight lately." c. "I wake up coughing every night." d. "I have trouble catching my breath."

ANS: B Signs of systemic congestion occur with right-sided heart failure. Fluid is retained, pressure builds in the venous system, and peripheral edema develops. Left-sided heart failure symptoms include respiratory symptoms. Orthopnea, coughing, and difficulty breathing all could be results of left-sided heart failure.

A client is scheduled to have a tracheostomy placed in an hour. What action by the nurse is the priority? a. Administer prescribed anxiolytic medication. b. Ensure that informed consent is on the chart. c. Reinforce any teaching done previously. d. Start the preoperative antibiotic infusion.

ANS: B Since this is an operative procedure, the client must sign an informed consent, which must be on the chart. Giving anxiolytics and antibiotics and reinforcing teaching may also be required but do not take priority.

A nurse cares for a client recovering from prosthetic valve replacement surgery. The client asks, "Why will I need to take anticoagulants for the rest of my life?" What is the best response by the nurse? a. "The prosthetic valve places you at greater risk for a heart attack." b. "Blood clots form more easily in artificial replacement valves." c. "The vein taken from your leg reduces circulation in the leg." d. "The surgery left a lot of small clots in your heart and lungs."

ANS: B Synthetic valve prostheses and scar tissue provide surfaces on which platelets can aggregate easily and initiate the formation of blood clots. The other responses are inaccurate.

A nurse cares for a client who has an 80% blockage of the right coronary artery (RCA) and is scheduled for bypass surgery. Which intervention would the nurse be prepared to implement while this client waits for surgery? a. Administration of IV furosemide b. Initiation of an external pacemaker c. Assistance with endotracheal intubation d. Placement of central venous access

ANS: B The RCA supplies the right atrium, right ventricle, inferior portion of the left ventricle, and atrioventricular (AV) node. It also supplies the sinoatrial node in 50% of people. If the client totally occludes the RCA, the AV node would not function and the client would go into heart block, so emergency pacing would be available for the client. Furosemide, intubation, and central venous access will not address the primary complication of RCA occlusion, which is AV node (and possibly SA node) malfunction.

After teaching a client who has an implantable cardioverter-defibrillator (ICD), a nurse assesses the client's understanding. Which statement by the client indicates correct understanding of the teaching? a. "I would wear a snug-fitting shirt over the ICD." b. "I will avoid sources of strong electromagnetic fields." c. "I would participate in a strenuous exercise program." d. "Now I can discontinue my antidysrhythmic medication."

ANS: B The client being discharged with an ICD is instructed to avoid strong sources of electromagnetic fields, such as devices emitting microwaves (not microwave ovens); transformers; radio, television, and radar transmitters; large electrical generators; metal detectors, including handheld security devices at airports; antitheft devices; arc welding equipment; and sources of 60-cycle (Hz) interference. Also avoid leaning directly over the alternator of a running motor of a car or boat. Clients would avoid tight clothing, which could cause irritation over the ICD generator. The client would be encouraged to exercise but would not engage in strenuous activities that cause the heart rate to meet or exceed the ICD cutoff point because the ICD can discharge inappropriately. The client would continue all prescribed medications.

A client with a new tracheostomy is being seen in the oncology clinic. What finding by the nurse best indicates that goals for the client's decrease in self-esteem are being met? a. The client demonstrates good understanding of stoma care. b. The client has joined a book club that meets at the library. c. Family members take turns assisting with stoma care. d. Skin around the stoma is intact without signs of infection.

ANS: B The client joining a book club that meets outside the home and requires him or her to go out in public is the best sign that goals for disrupted self-esteem are being met. The other findings are all positive signs but do not relate to this client problem.

A nurse teaches a client who had a supraglottic laryngectomy. Which technique would the nurse teach the client to prevent aspiration? a. Tilt the head back as far as possible when swallowing. b. Swallow twice while bearing down. c. Breathe slowly and deeply while swallowing. d. Keep the head very still and straight while swallowing.

ANS: B The client post supraglottic laryngectomy has a high risk for aspiration. The nurse or speech language pathologist teaches the client the supraglottic method of swallowing. This includes placing a small amount of food in the mouth, performing the Valsalva maneuver, then swallowing twice. The client sits upright. The client holds the breath while swallowing twice. Keeping the head still and straight will not decrease the risk of aspiration.

A pulmonary nurse cares for clients who have chronic obstructive pulmonary disease (COPD). Which client would the nurse assess first? a. A 46 year old with a 30-pack-year history of smoking b. A 52 year old in a tripod position using accessory muscles to breathe c. A 68 year old who has dependent edema and clubbed fingers d. A 74 year old with a chronic cough and thick, tenacious secretions

ANS: B The client who is in a tripod position and using accessory muscles is working to breathe. This client must be assessed first to establish how effectively the client is breathing and provide interventions to minimize respiratory distress. The other clients are not in acute distress.

A nurse assesses a client with pericarditis. Which assessment finding would the nurse expect to find? a. Heart rate that speeds up and slows down. b. Friction rub at the left lower sternal border. c. Presence of a regular gallop rhythm. d. Coarse crackles in bilateral lung bases.

ANS: B The client with pericarditis may present with a pericardial friction rub at the left lower sternal border. This sound is the result of friction from inflamed pericardial layers when they rub together. The other assessments are not related.

The nurse is evaluating a 3-day diet history with a client who has an elevated lipid panel. What meal selection indicates that the client is managing this condition well with diet? a. A 4-ounce steak, French fries, iceberg lettuce b. Baked chicken breast, broccoli, tomatoes c. Fried catfish, cornbread, peas d. Spaghetti with meat sauce, garlic bread

ANS: B The diet recommended for this client would be low in saturated fats and red meat, high in vegetables and whole grains (fiber), low in salt, and low in trans fat. The best choice is the chicken with broccoli and tomatoes. The French fries have too much fat and the iceberg lettuce has little fiber. The catfish is fried. The spaghetti dinner has too much red meat and no vegetables.

A nurse cares for a client who is prescribed magnetic resonance imaging (MRI) of the heart. The client's health history includes a previous myocardial infarction and pacemaker implantation. What action would the nurse take? a. Schedule an electrocardiogram just before the MRI. b. Notify the primary health care provider before scheduling the MRI. c. Request lab for cardiac enzymes from the primary health care provider. d. Instruct the client to increase fluid intake the day before the MRI.

ANS: B The magnetic fields of the MRI can deactivate the pacemaker. The nurse would call the primary health care provider and report that the client has a pacemaker so that he or she can order other diagnostic tests. The client does not need an electrocardiogram, cardiac enzymes, or increased fluids. Some newer MRI scanners have eliminated the possibility of complications due to implants, but the nurse needs to notify the primary health care provider.

1A nurse observes that a client's anteroposterior (AP) chest diameter is the same as the lateral chest diameter. Which question would the nurse ask the client in response to this finding? a. "Are you taking any medications or herbal supplements?" b. "Do you have any chronic breathing problems?" c. "How often do you perform aerobic exercise?" d. "What is your occupation and what are your hobbies?"

ANS: B The normal chest has an anteroposterior (AP or front-to-back) diameter ratio with the lateral (side-to-side) diameter. This ratio normally is about 1:1.5. When the AP diameter approaches the lateral diameter, and the ratio is 1:1, the client is said to have a barrel chest. Most commonly, barrel chest occurs as a result of a long-term chronic airflow limitation problem, such as chronic emphysema. It can also be seen in people who have lived at a high altitude for many years. Medications, herbal supplements, and aerobic exercise are not associated with a barrel chest. Although occupation and hobbies may expose a client to irritants that can cause chronic lung disorders and barrel chest, asking about chronic breathing problems is more direct and would be asked first.

A nurse is caring for four client s. Which client would the nurse assess first? a. Client with an acute myocardial infarction, pulse 102 beats/min b. Client who is 1 hour post-angioplasty, and has tongue swelling and anxiety c. Client who is post coronary artery bypass, with chest tube drained 100 mL/hr d. Client who is post coronary artery bypass, with potassium 4.2 mEq/L (4.2 mmol/L)

ANS: B The post-angioplasty client with tongue swelling and anxiety is exhibiting signs and symptoms of an allergic reaction (perhaps to the contrast medium) that could progress to anaphylaxis. The nurse would assess this client first. The client with a heart rate of 102 beats/min may have increased oxygen demands but is just over the normal limit for heart rate. The two post coronary artery bypass clients are stable.

A nurse answers a call light and finds a client anxious, short of breath, reporting chest pain, and has a blood pressure of 88/52 mm Hg. What action by the nurse takes priority? a. Assess the client's lung sounds. b. Notify the Rapid Response Team. c. Provide reassurance to the client. d. Take a full set of vital signs.

ANS: B This client has signs and symptoms of a pulmonary embolism, and the most critical action is to notify the Rapid Response Team for speedy diagnosis and treatment. The other actions are appropriate also but are not the priority.

A client seen in the emergency department reports fever, fatigue, and dry cough but no other upper respiratory symptoms. A chest x-ray reveals mediastinal widening. What action by the nurse is best? a. Collect a sputum sample for culture by deep suctioning. b. Inform the client that oral antibiotics will be needed for 60 days. c. Place the client on Airborne Precautions immediately. d. Tell the client that directly observed therapy is needed.

ANS: B This client has signs and symptoms of early inhalation anthrax. For treatment, after IV antibiotics are finished, oral antibiotics are continued for at least 60 days. Sputum cultures are not needed. Anthrax is not transmissible from person to person, so Standard Precautions are adequate. Directly observed therapy is often used for tuberculosis.

6. A nurse is caring for a client on the medical stepdown unit. The following data are related to this client: Subjective Information Shortness of breath for 20 minutes Reports feeling frightened "Can't catch my breath" Laboratory Analysis pH: 7.32 PaCO2: 28 mm Hg PaO2: 78 mm Hg SaO2: 88% Physical Assessment Pulse: 120 beats/min Respiratory rate: 34 breaths/min Blood pressure 158/92 mm Hg Lungs have crackles What action by the nurse is most appropriat? a. Call respiratory therapy for a breathing treatment. b. Facilitate a STAT pulmonary angiography. c. Prepare for immediate endotracheal intubation. d. Prepare to administer intravenous anticoagulants.

ANS: B This client has signs and symptoms of pulmonary embolism (PE); however, many conditions can cause the client's presentation. The gold standard for diagnosing a PE is pulmonary angiography. The nurse would facilitate this test as soon as possible. The client does not have wheezing, so a respiratory treatment is not needed. The client is not unstable enough to need intubation and mechanical ventilation. IV anticoagulants are not given without a diagnosis of PE.

A client is in the hospital after suffering a myocardial infarction and has bathroom privileges. The nurse assists the client to the bathroom and notes the client's O2 saturation to be 95%, pulse 88 beats/min, and respiratory rate 16 breaths/min after returning to bed. What action by the nurse is best? a. Administer oxygen at 2 L/min. b. Allow continued bathroom privileges. c. Obtain a bedside commode. d. Suggest the client use a bedpan.

ANS: B This client's physiologic parameters did not exceed normal during and after activity, so it is safe for the client to continue using the bathroom. There is no indication that the client needs oxygen, a commode, or a bedpan.

A nurse is caring for four clients on intravenous heparin therapy. Which laboratory value possibly indicates that a serious side effect has occurred? a. Hemoglobin: 14.2 g/dL (142 g/L) b. Platelet count: 82,000/L (82 109/L) c. Red blood cell count: 4.8/mm3 (4.8 1012/L) d. White blood cell count: 8700/mm3 (8.7 109/L)

ANS: B This platelet count is low and could indicate heparin-induced thrombocytopenia. The other values are normal for either gender.

After teaching a client how to perform diaphragmatic breathing, the nurse assesses the client's understanding. Which action demonstrates that the client correctly understands the teaching? a. The client lies on his or her side with knees bent. b. The client places his or her hands on the abdomen. c. The client lies in a prone position with straight. d. The client places his or her hands above the head.

ANS: B To perform diaphragmatic breathing correctly, the client would place his or her hands on the abdomen to create resistance. This type of breathing cannot be performed effectively while lying on the side or with hands over the head. This type of breathing would not be as effective lying prone.

A client has a deep vein thrombosis (DVT). What comfort measure does the nurse delegate to the assistive personnel (AP)? a. Ambulate the client. b. Apply a warm moist pack. c. Massage the client's leg. d. Provide an ice pack.

ANS: B Warm moist packs will help with the pain of a DVT. Ambulation is not a comfort measure. Massaging the client's legs is contraindicated to prevent complications such as pulmonary embolism. Ice packs are not recommended for DVT.

A nurse obtains the health history of a client who is newly admitted to the medical unit. Which statement by the client would alert the nurse to the presence of edema? a. "I wake up to go to the bathroom at night." b. "My shoes fit tighter by the end of the day." c. "I seem to be feeling more anxious lately." d. "I drink at least eight glasses of water a day."

ANS: B Weight gain can result from fluid accumulation in the interstitial spaces. This is known as edema. The nurse would note whether the client feels that his or her shoes or rings are tight, and would observe, when present, an indentation around the leg where the socks end. The other answers do not describe edema.

A client in the emergency department is taking rifampin for tuberculosis. The client reports yellowing of the sclera and skin and bleeding after minor trauma. What laboratory results correlate to this condition? (Select all that apply.) a. Blood urea nitrogen (BUN): 19 mg/dL (6.7 mmol/L) b. International normalized ratio (INR): 6.3 c. Prothrombin time: 35 seconds d. Serum sodium: 130 mEq/L (130 mmol/L) e. White blood cell (WBC) count: 72,000/mm3 (72 109/L)

ANS: B, C Rifampin can cause liver damage, evidenced by the client's high INR and prothrombin time. The BUN and WBC count are normal. The sodium level is low, but that is not related to this client's problem.

What nonpharmacologic comfort measures should the nurse include in the plan of care for a client with severe varicose veins? (Select all that apply.) a. Administering mild analgesics for pain b. Applying elastic compression stockings c. Elevating the legs when sitting or lying d. Reminding the client to do leg exercises e. Teaching the client about surgical options

ANS: B, C, D The three E's of care for varicose veins include elastic compression hose, exercise and elevation. Mild analgesics are not a nonpharacologic measure. Teaching about surgical options is not a comfort measure. High impact anaerobic are not encouraged and are not a comfort measure.

A nurse learns about modifiable risk factors for coronary artery disease. Which factors does this include? (Select all that apply.) a. Age b. Hypertension c. Obesity d. Smoking e. Stress f. Gender

ANS: B, C, D, E Hypertension, obesity, smoking, and excessive stress are all modifiable risk factors for coronary artery disease. Age and gender are not nonmodifiable risk factors.

A nurse prepares to discharge a client who has heart failure. Based on national quality measures, what actions would the nurse complete prior to discharging this client? (Select all that apply.) a. Teach the client about energy conservation techniques. b. Ensure that the client is prescribed a beta blocker. c. Document a discussion about advanced directives. d. Confirm that a postdischarge nurse visit has been scheduled. e. Consult a social worker for additional resources. f. Care transition record transmitted to next level of care within 7 days of discharge.

ANS: B, C, D, F National quality measures aim to decrease heart failure readmission by proper preparation for discharge. These measures include :(1) beta blocker prescribed for left ventricular dysfunction at discharge, (2) postdischarge follow-up appointment scheduled within 7 days of discharge with documentation of location, date, and time. (3) care transition record transmitted to next level of care within 7 days of discharge. (4) documentation of discussion of advance directives/advance care planning with a health care provider, (5) documentation of execution of advance directives within the medical record, and (6) postdischarge evaluation of patient for symptom assessment and treatment adherence within 72 hours of discharge (this can occur by phone, scheduled office visit, or home visit)

A nurse prepares a client who is scheduled for a bronchoscopy with transbronchial biopsy procedure at 9:00 AM (0900). What actions would the nurse take? (Select all that apply.) a. Provide a clear liquid breakfast. b. Verify that the informed consent was obtained. c. Document the client's allergies. d. Review laboratory results. e. Hold the client's bronchodilator. f. Monitor the client for at least 24 hours afterwards.

ANS: B, C, D, F Prior to a bronchoscopy, the nurse would verify that the informed consent was obtained, keep the client NPO for 4 to 8 hours prior to the procedure or per agency policy to prevent aspiration, document allergies, and review laboratory results including complete blood count and bleeding times. There is no reason to hold the client's bronchodilator prior to this procedure. The nurse will monitor the client at least every 4 hours for 24 hours.

A nurse is teaching a client how to perform pursed-lip breathing. Which instructions would the nurse include in this teaching? (Select all that apply.) a. "Open your mouth and breathe deeply." b. "Use your abdominal muscles to squeeze air out of your lungs." c. "Breath out slowly without puffing your cheeks." d. "Focus on inhaling and holding your breath as long as you can." e. "Exhale at least twice the amount of time it took to breathe in." f. "Lie on your back with your knees bent."

ANS: B, C, E A nurse would teach a client to close his or her mouth and breathe in through his or her nose, purse his or her lips and breathe out slowly without puffing his or her cheeks, and use his or her abdominal muscles to squeeze out every bit of air. The nurse would also remind the client to use pursed-lip breathing during any physical activity, to focus on exhaling, and to never hold his or her breath. Lying on the back with bent knees is the preferred position for diaphragmatic breathing.

An emergency department nurse assesses a female client. Which assessment findings would alert the nurse to request a prescription for an electrocardiogram? (Select all that apply.) a. Hypertension b. Fatigue despite adequate rest c. Indigestion d. Abdominal pain e. Shortness of breath

ANS: B, C, E Women may not have chest pain with myocardial infarction, but may feel discomfort or indigestion. They often present with a triad of symptoms—indigestion or feeling of abdominal fullness, feeling of chronic fatigue despite adequate rest, and feeling unable to catch their breath. Frequently, women are not diagnosed and therefore are not treated adequately. Hypertension and abdominal pain are not associated with acute coronary syndrome.

The nurse is teaching a client with obstructive sleep apnea (OSA) about the prescribed CPAP. What information does the nurse include? (Select all that apply.) a. Insurance will cover the cost if you wear it at least 4 hours a day. b. Once the delivery mask is adjusted, do not loosen the straps. c. The CPAP provides pressure that holds your upper airways open. d. You need to clean the mask at least once a week to prevent infection. e. The humidification increases the risk of fungal infections. f. Be patient when first using the system, it can be frustrating at first.

ANS: B, C, E, F A CPAP for OSA provides pressure that keeps the upper airway open. A properly fitting mask or nasal pillows is necessary to provide the pressure. Humidification in the system leads to an increased risk for fungal infections. Patients may have anxiety about using the equipment and worry about it being disruptive; most clients have a period of adjustment when first starting to use a CPAP. Medicare will usually cover the cost if the client wears the CPAP at least 6 hours a day. The mask or pillows should be cleaned daily.

A nurse assesses a client with chronic obstructive pulmonary disease. Which questions would the nurse ask to determine the client's activity tolerance? (Select all that apply.) a. "What color is your sputum?" b. "Do you have any difficulty sleeping?" c. "How long does it take to perform your morning routine?" d. "Do you walk upstairs every day?" e. "Have you lost any weight lately?" f. "How does your activity compare to this time last year?"

ANS: B, C, E, F Difficulty sleeping could indicate worsening breathlessness, as could taking longer to perform activities of daily living. Weight loss could mean increased dyspnea as the client becomes too fatigued to eat. The color of the client's sputum would not assist in determining activity tolerance. Asking whether the client walks upstairs every day is not as pertinent as determining if the client becomes short of breath on walking upstairs, or if the client goes upstairs less often than previously. The nurse would ask the client to compare his or her current level of activity with that of a month or even a year ago.

Prior to discharge, a client who had an acute myocardial infarction and coronary artery bypass graft asks the nurse about sexual activity. What information does the nurse provide? (Select all that apply.) a. "You will need to wait at least 6 weeks before intercourse.' b. "Your usual sexual activity is not likely to damage your heart." c. "Start having sex when you are most rested, like in the morning." d. "When you can climb four flights of stairs, you can tolerate sex." e. "Don't eat for three hours before engaging in sexual activity." f. "Use a comfortable position that doesn't stress your incision."

ANS: B, C, F Clients have many concerns about resuming sexual activity after an acute coronary event. Generally, once the client can walk one block or climb two flights of stairs, he or she can tolerate sex. The client should start after a period of rest and at least 11/2 hours after a heavy meal or exercise. Clients should be taught to choose a position that is comfortable for both parties and does not place undue stress on their incisions or on their hearts.

A nurse assesses a client who is recovering after a coronary catheterization. Which assessment findings in the first few hours after the procedure require immediate action by the nurse? (Select all that apply.) a. Blood pressure of 140/88 mm Hg b. Serum potassium of 2.9 mEq/L (2.9 mmol/L) c. Warmth and redness at the site d. Expanding groin hematoma e. Rhythm changes on the cardiac monitor f. Oxygen saturation 93% on room air

ANS: B, D, E After a cardiac catheterization, the nurse monitors vital signs, entry site, cardiac function, and distal circulation. The potassium is very low which can lead to dysrhythmias. An expanding hematoma signifies bleeding. Rhythm changes on the monitor are a known complication. These findings would require prompt action. The client's blood pressure is slightly elevated but does not need immediate action. Warmth and redness at the site would indicate an infection, but this would not be present in the first few hours. The oxygen saturation is slightly low but not critical and there is no baseline to compare it to.

A nurse prepares a client for a pharmacologic stress echocardiogram. What actions would the nurse take when preparing this client for the procedure? (Select all that apply.) a. Assist the primary health care provider to place a central venous access device. b. Prepare for continuous blood pressure and pulse monitoring. c. Administer the client's prescribed beta blocker. d. Give the client nothing by mouth 3 to 6 hours before the procedure. e. Explain to the client that dobutamine will simulate exercise for this examination.

ANS: B, D, E Clients receiving a pharmacologic stress echocardiogram will need peripheral venous access and continuous blood pressure and pulse monitoring. The client must be NPO 3 to 6 hours prior to the procedure. Education about dobutamine, which will be administered during the procedure, would be performed. Beta blockers are often held prior to the procedure as they lower the heart rate and may result in inaccurate results.

A nurse is caring for five clients. For which clients would the nurse assess a high risk for developing a pulmonary embolism (PE)? (Select all that apply.) a. Client who had a reaction to contrast dye yesterday b. Client with a new spinal cord injury on a rotating bed c. Middle-age client with an exacerbation of asthma d. Older client who is 1 day post-hip replacement surgery e. Young obese client with a fractured femur f. Middle-age adult with a history of deep vein thrombosis

ANS: B, D, E Conditions that place clients at higher risk of developing PE include prolonged immobility, central venous catheters, surgery, obesity, advancing age, conditions that increase blood clotting, history of thromboembolism, smoking, pregnancy, estrogen therapy, heart failure, stroke, cancer (particularly lung or prostate), and trauma. A contrast dye reaction and asthma pose no risk for PE.

A client is 1-day postoperative after a coronary artery bypass graft. What nonpharmacologic comfort measures does the nurse include when caring for this client? (Select all that apply.) a. Administer pain medication before ambulating. b. Assist the client into a position of comfort in bed. c. Encourage high-protein diet selections. d. Provide complementary therapies such as music. e. Remind the client to splint the incision when coughing.

ANS: B, D, E Nonpharmacologic comfort measures can include positioning, complementary therapies, and splinting the chest incision. Medications are not nonpharmacologic. Food choices are not comfort measures.

A nurse assesses a client who is recovering from a thoracentesis. Which assessment findings would alert the nurse to a potential pneumothorax? (Select all that apply.) a. Bradycardia b. New-onset cough c. Purulent sputum d. Tachypnea e. Pain with respirations f. Rapid, shallow respirations

ANS: B, D, E Symptoms of a pneumothorax include tachycardia, tachypnea, new-onset "nagging" cough, and pain that is worse at the end of inhalation and the end of exhalation on the affected side. Additional symptoms include trachea slanted to the unaffected side, cyanosis, and the affected side of the chest that does not move in and out with respirations. Purulent sputum is a symptom of infection.

A client with a known abdominal aortic aneurysm reports dizziness and severe abdominal pain. The nurse assesses the client's blood pressure at 82/40 mm Hg. What actions by the nurse are most important? (Select all that apply.) a. Administer pain medication. b. Assess distal pulses every 10 minutes. c. Have the client sign a surgical consent. d. Notify the Rapid Response Team. e. Take vital signs every 10 minutes.

ANS: B, D, E This client may have a ruptured/rupturing aneurysm. The nurse would notify the Rapid Response team and perform frequent client assessments. Giving pain medication will lower the client's blood pressure even further. The nurse cannot have the client sign a consent until the surgeon has explained the procedure.

A nurse assesses a client who has a mediastinal chest tube. Which symptoms require the nurse's immediate intervention? (Select all that apply.) a. Production of pink sputum b. Tracheal deviation c. Pain at insertion site d. Sudden onset of shortness of breath e. Drainage greater than 70 mL/hr f. Disconnection at Y site

ANS: B, D, E, F Immediate intervention is warranted if the client has tracheal deviation because this could indicate a tension pneumothorax. Sudden shortness of breath could indicate dislodgment of the tube, occlusion of the tube, or pneumothorax. Drainage greater than 70 mL/hr could indicate hemorrhage. Disconnection at the Y site could result in air entering the tubing. Production of pink sputum and pain at the insertion site are not signs/symptoms that would require immediate intervention.

A client presents to the emergency department with a thoracic aortic aneurysm. Which findings are most consistent with this condition? (Select all that apply.) a. Abdominal tenderness b. Difficulty swallowing c. Changes in bowel habits d. Shortness of breath e. Hoarseness

ANS: B, E Signs of a thoracic aortic aneurysm include shortness of breath, hoarseness, and difficulty swallowing. Pain is often rated as a 10 on a 10-point scale. Bowel habits are not related.

A client is receiving an infusion of tissue plasminogen activator (tPA). The nurse assesses the client to be disoriented to person, place, and time. What action by the nurse is best? a. Assess the client's pupillary responses. b. Request a neurologic consultation. c. Call the primary health care provider immediately. d. Take and document a full set of vital signs.

ANS: C A change in neurologic status in a client receiving t-PA could indicate intracranial hemorrhage. The nurse would notify the primary health care provider immediately. A full assessment, including pupillary responses and vital signs, occurs next. The nurse may or may not need to call a neurologist.

A telemetry nurse assesses a client who has a heart rate of 35 beats/min on the cardiac monitor. Which assessment would the nurse complete next? a. Pulmonary auscultation b. Pulse strength and amplitude c. Level of consciousness d. Mobility and gait stability

ANS: C A heart rate of 40 beats/min or less could have hemodynamic consequences. The client is at risk for inadequate cerebral perfusion. The nurse would assess for level of consciousness, dizziness, confusion, syncope, chest pain, shortness of breath. Although the other assessments would be completed, the nurse would assess the client's neurologic status next.

A client has a large pulmonary embolism and is started on oxygen. The nurse asks the charge nurse why the client's oxygen saturation has not significantly improved. What response by the nurse is best? a. "Breathing so rapidly interferes with oxygenation." b. "Maybe the client has respiratory distress syndrome." c. "The blood clot interferes with perfusion in the lungs." d. "The client needs immediate intubation and mechanical ventilation."

ANS: C A large blood clot in the lungs will significantly impair gas exchange and oxygenation. Unless the clot is dissolved, this process will continue unabated. Hyperventilation can interfere with oxygenation by shallow breathing, but there is no evidence that the client is hyperventilating, and this is also not the most precise physiologic answer. Acute respiratory distress syndrome can occur, but this is not as likely soon after the client starts on oxygen plus there is no indication of how much oxygen the client is on. The client may need to be mechanically ventilated, but without concrete data on FiO2 and SaO2, the nurse cannot make that judgment.

A nurse assesses a client who is recovering after a left-sided cardiac catheterization. Which assessment finding requires immediate intervention? a. Urinary output less than intake b. Bruising at the insertion site c. Slurred speech and confusion d. Discomfort in the left leg

ANS: C A left-sided cardiac catheterization specifically increases the risk for a cerebral vascular accident. A change in neurologic status needs to be acted on immediately. Discomfort and bruising are not unexpected at the site. Urinary output less than intake may or may not be significant.

1. A nurse assesses a client after an open lung biopsy. Which assessment finding is matched with the correct intervention? a. Client reports being dizzy—nurse calls the Rapid Response Team. b. Client's heart rate is 55 beats/min—nurse withholds pain medication. c. Client has reduced breath sounds—nurse calls primary health care provider immediately. d. Client's respiratory rate is 18 breaths/min—nurse decreases oxygen flow rate.

ANS: C A potentially serious complication after biopsy is pneumothorax, which is indicated by decreased or absent breath sounds. The primary health care provider needs to be notified immediately. Dizziness without other data would not lead the nurse to call the RRT. If the client's heart rate is 55 beats/min, no reason is known to withhold pain medication. A respiratory rate of 18 breaths/min is a normal finding and would not warrant changing the oxygen flow rate.

A client in the cardiac stepdown unit reports severe, crushing chest pain accompanied by nausea and vomiting. What action by the nurse takes priority? a. Administer an aspirin. b. Call for an electrocardiogram (ECG). c. Maintain airway patency. d. Notify the provider.

ANS: C Airway always is the priority. The other actions are important in this situation as well, but the nurse would stay with the client and ensure that the airway remains patent (especially if vomiting occurs) while another person calls the primary health care provider (or Rapid Response Team) and facilitates getting an ECG done. Aspirin will probably be administered, depending on the primary health care provider's prescription and the client's current medications.

A nurse cares for a client who tests positive for alpha1-antitrypsin (AAT) deficiency. The client asks, "What does this mean?" How would the nurse respond? a. "Your children will be at high risk for chronic obstructive pulmonary disease." b. "I will contact a genetic counselor to discuss your condition." c. "Your risk for chronic obstructive pulmonary disease is higher, especially if you smoke." d. "This is a recessive gene and would have no impact on your health."

ANS: C Alpha1-antitrypsin deficiency is an important risk factor for COPD. The gene for AAT is a recessive gene. Clients with only one allele produce enough AAT to prevent COPD unless the client smokes or there is sufficient exposure to other inhalants. A client with two alleles is at high risk for COPD even if not exposed to smoke or other irritants. The client is a carrier, and children may or may not be at high risk depending on the partner's AAT levels. Contacting a genetic counselor may be helpful but does not address the client's current question.

A nurse cares for a client who is infected with Burkholderia cepacia. What action would the nurse take first when admitting this client to a pulmonary care unit? a. Instruct the client to wash his or her hands after contact with other people. b. Implement Droplet Precautions and don a surgical mask. c. Keep the client separated from other clients with cystic fibrosis. d. Obtain blood, sputum, and urine culture specimens.

ANS: C B. cepacia infection is spread through casual contact between cystic fibrosis clients, thus the need for infected clients to be separated from noninfected clients. Strict isolation measures will not be necessary. Although the client would wash his or her hands frequently, the most important measure that can be implemented on the unit is isolation of the client from other clients with cystic fibrosis. There is no need to implement Droplet Precautions or don a surgical mask when caring for this client. Obtaining blood, sputum, and urine culture specimens will not provide information necessary to care for a client with B. cepacia infection.

After teaching a client who is recovering from a heart transplant to change positions slowly, the client asks, "Why is this important?" How would the nurse respond? a. "Rapid position changes can create shear and friction forces, which can tear out your internal vascular sutures." b. "Your new vascular connections are more sensitive to position changes, leading to increased intravascular pressure and dizziness." c. "Your new heart is not connected to the nervous system and is unable to respond to decreases in blood pressure caused by position changes." d. "While your heart is recovering, blood flow is diverted away from the brain, increasing the risk for stroke when you stand up."

ANS: C Because the new heart is denervated, the baroreceptor and other mechanisms that compensate for blood pressure drops caused by position changes do not function. This allows orthostatic hypotension to persist in the postoperative period. The other options are false statements and do not correctly address the client's question.

A nurse administers prescribed adenosine to a client. Which response would the nurse assess for as the expected therapeutic response? a. Decreased intraocular pressure b. Increased heart rate c. Short period of asystole d. Hypertensive crisis

ANS: C Clients usually respond to adenosine with a short period of asystole, bradycardia with long pauses, nausea, or vomiting. Adenosine has no impact on intraocular pressure nor does it cause increased heart rate or hypertensive crisis.

A nurse cares for a client who is recovering from a myocardial infarction. The client states, "I will need to stop eating so much chili to keep that indigestion pain from returning." What is the nurse's best response? a. "Chili is high in fat and calories; it would be a good idea to stop eating it." b. "The primary health care provider has prescribed an antacid every morning." c. "What do you understand about what happened to you?" d. "When did you start experiencing this indigestion?"

ANS: C Clients who experience myocardial infarction often respond with denial, which is a defense mechanism. The nurse would ask the client what he or she thinks happened, or what the illness means to him or her. The other responses do not address the client's misconception about recent pain and the cause of that pain.

A nurse cares for a client with atrial fibrillation who reports fatigue when completing activities of daily living. What intervention would the nurse implement to address this client's concerns? a. Administer oxygen therapy at 2 L per nasal cannula. b. Provide the client with a sleeping pill to stimulate rest. c. Schedule periods of exercise and rest during the day. d. Ask assistive personnel (AP) to help bathe the client.

ANS: C Clients who have atrial fibrillation are at risk for decreased cardiac output and fatigue when completing activities of daily living. The nurse would schedule periods of exercise and rest during the day to decrease fatigue. The other interventions will not assist the client with performing self-care activities and there is no indication for oxygen.

A nurse teaches a client who is prescribed nicotine replacement therapy. Which statement would the nurse include in this client's teaching? a. "Make a list of reasons why smoking is a bad habit." b. "Rise slowly when getting out of bed in the morning." c. "Smoking while taking this medication will increase your risk of a stroke." d. "Stopping this medication suddenly increases your risk for a heart attack."

ANS: C Clients who smoke while using drugs for nicotine replacement therapy increase the risk of stroke and heart attack. Nurses would teach clients not to smoke while taking these drugs. The nurse would encourage the client to make a list of reasons for stopping the habit but would not phrase it so judgmentally. Orthostatic hypotension is not a risk with nicotine replacement therapy. Stopping suddenly does not increase the risk of heart attack.

The nurse asks a client who has experienced ventricular dysrhythmias about substance abuse. The client asks, "Why do you want to know if I use cocaine?" What is the nurse's best response? a. "Substance abuse puts clients at risk for many health issues." b. "The hospital requires that I ask you about cocaine use." c. "Clients who use cocaine are at risk for fatal dysrhythmias." d. "We can provide services for cessation of substance abuse."

ANS: C Clients who use cocaine or illicit inhalants are particularly at risk for potentially fatal dysrhythmias. The other responses do not adequately address the client's question.

A nurse is teaching a client who has cystic fibrosis (CF). Which statement would the nurse include in this client's teaching? a. "Take an antibiotic each day." b. "You should get genetic screening." c. "Eat a well-balanced, nutritious diet." d. "Plan to exercise for 30 minutes every day."

ANS: C Clients with CF often are malnourished due to vitamin deficiency and pancreatic malfunction. Maintaining nutrition is essential. Daily antibiotics and daily exercise are not essential actions. Genetic screening might be an option; however, the nurse would not just tell the client to do something like that.

A nurse assesses a client in an outpatient clinic. Which statement alerts the nurse to the possibility of left-sided heart failure? a. "I have been drinking more water than usual." b. "I am awakened by the need to urinate at night." c. "I must stop halfway up the stairs to catch my breath." d. "I have experienced blurred vision on several occasions."

ANS: C Clients with left-sided heart failure report weakness or fatigue while performing normal activities of daily living, as well as difficulty breathing, or "catching their breath." This occurs as fluid moves into the alveoli. Nocturia is often seen with right-sided heart failure. Thirst and blurred vision are not related to heart failure.

A nurse cares for a client with chronic obstructive pulmonary disease (COPD) who appears thin and disheveled. Which question would the nurse ask first? a. "Do you have a strong support system?" b. "What do you understand about your disease?" c. "Do you experience shortness of breath with basic activities?" d. "What medications are you prescribed to take each day?"

ANS: C Clients with severe COPD may not be able to perform daily activities, including bathing and eating, because of excessive shortness of breath. The nurse would ask the client if shortness of breath is interfering with basic activities. Although the nurse would need to know about the client's support systems, current knowledge, and medications, these questions do not address the client's appearance.

A nurse is assessing an obese client in the clinic for follow-up after an episode of deep vein thrombosis. The client has lost 20 lb (9.09 Kg) since the last visit. What action by the nurse is best? a. Ask if the weight loss was intended. b. Encourage a high-protein, high-fiber diet. c. Measure for new compression stockings. d. Review a 3-day food recall diary.

ANS: C Compression stockings must fit correctly in order to work. After losing a significant amount of weight, the client would be remeasured and new stockings ordered if needed. The other options are appropriate, but not the most important.

A nurse cares for a client who has a family history of cystic fibrosis. The client asks, "Will my children have cystic fibrosis?" How would the nurse respond? a. "Since many of your family members are carriers, your children will also be carriers of the gene." b. "Cystic fibrosis is an autosomal recessive disorder. If you are a carrier, your children will have the disorder." c. "Since you have a family history of cystic fibrosis, I would encourage you and your partner to be tested." d. "Cystic fibrosis is caused by a protein that controls the movement of chloride. Adjusting your diet will decrease the spread of this disorder."

ANS: C Cystic fibrosis is an autosomal recessive disorder in which both gene alleles must be mutated for the disorder to be expressed. The nurse would encourage both the client and partner to be tested for the abnormal gene. The other statements are not true.

A nurse is assisting the primary health care provider (PHCP) who is intubating a client. The PHCP has been attempting to intubate for 40 seconds. What action by the nurse is best? a. Ensure that the client has adequate sedation. b. Find another qualified provider to intubate. c. Interrupt the procedure to give oxygen. d. Monitor the client's oxygen saturation.

ANS: C Each intubation attempt should not exceed 30 seconds (15 is preferable) as it causes hypoxia. The nurse would interrupt the intubation attempt and give the client oxygen. The nurse would also have adequate sedation during the procedure and monitor the client's oxygen saturation, but these do not take priority. Finding another qualified provider to intubate the client is not appropriate at this time.

A client asks what "essential hypertension" is. What response by the registered nurse is best? a. "It means it is caused by another disease." b. "It means it is 'essential' that it be treated." c. "It is hypertension with no specific cause." d. "It refers to severe and life-threatening hypertension."

ANS: C Essential hypertension is the most common type of hypertension and has no specific cause such as an underlying disease process. Hypertension that is due to another disease process is called secondary hypertension. A severe, life-threatening form of hypertension is malignant hypertension.

A client is admitted with a pulmonary embolism (PE). The client is young, healthy, and active and has no known risk factors for PE. What action by the nurse is most appropriate? a. Encourage the client to walk 5 minutes each hour. b. Refer the client to smoking cessation classes. c. Teach the client about factor V Leiden testing. d. Tell the client that sometimes no cause for disease is found.

ANS: C Factor V Leiden is an inherited thrombophilia that can lead to abnormal clotting events, including PE. A client with no known risk factors for this disorder would be asked about family history and referred for testing. Encouraging the client to walk is healthy, but is not related to the development of a PE in this case, nor is smoking. Although there are cases of disease where no cause is ever found, this assumption is premature.

A nurse working in a geriatric clinic sees clients with "cold" symptoms and rhinitis. The primary health care provider (PHCP) often leaves a prescription for diphenhydramine. What action by the nurse is best? a. Teach the client about possible drowsiness. b. Instruct the client to drink plenty of water. c. Consult with the PHCP about the medication. d. Encourage the client to take the medication with food.

ANS: C First-generation antihistamines are not appropriate for use in the older population. These drugs include chlorpheniramine, diphenhydramine, and hydroxyzine. The nurse would consult with the PHCP and request a different medication. Diphenhydramine does cause drowsiness, but the nurse would request a different medication. Drinking plenty of fluids is appropriate for the condition and is not related to the medication. Antihistamines can be taken without regard to food.

A nurse assesses female client who is experiencing a myocardial infarction. Which clinical manifestation would the nurse expect? a. Excruciating pain on inspiration b. Left lateral chest wall pain c. Fatigue and shortness of breath d. Numbness and tingling of the arm

ANS: C In women, fatigue, shortness of breath, and indigestion may be the major symptoms of myocardial infarction caused by poor cardiac output. Chest pain is the classic symptom of myocardial infarction and can be present in women. Pain on inspiration may be related to a pleuropulmonary cause. Numbness and tingling of the arm could also be related to the myocardial infarction, but are not known to be specific symptoms for women having and MI.

A nurse has educated a client on isoniazid. What statement by the client indicates that teaching has been effective? a. "I need to take extra vitamin C while on isoniazid." b. "I should take this medicine with milk or juice." c. "I will take this medication on an empty stomach." d. "My contact lenses will be permanently stained."

ANS: C Isoniazid needs to be taken on an empty stomach, either 1 hour before or 2 hours after meals. Extra vitamin B needs to be taken while on the drug. Staining of contact lenses commonly occurs while taking rifampin.

After teaching a client who is prescribed a long-acting beta2 agonist medication, a nurse assesses the client's understanding. Which statement indicates that the client comprehends the teaching? a. "I will carry this medication with me at all times in case I need it." b. "I will take this medication when I start to experience an asthma attack." c. "I will take this medication every morning to help prevent an acute attack." d. "I will be weaned off this medication when I no longer need it."

ANS: C Long-acting beta2 agonist medications will help prevent an acute asthma attack because they are long acting. The client will take this medication every day for best effect. The client does not have to always keep this medication with him or her because it is not used as a rescue medication. This is not the medication the client will use during an acute asthma attack because it does not have an immediate onset of action. The client will not be weaned off this medication because this is likely to be one of his or her daily medications.

A nurse assesses a client 2 hours after a cardiac angiography via the left femoral artery. The nurse notes that the left pedal pulse is weak. What action would the nurse take next? a. Elevate the leg and apply a sandbag to the entrance site. b. Increase the flow rate of intravenous fluids. c. Assess the color and temperature of the left leg. d. Document the finding as "left pedal pulse of +1/4."

ANS: C Loss of a pulse distal to an angiography entry site is serious, indicating a possible arterial obstruction. The left pulse would be compared with the right, and pulses would be compared with previous assessments, especially before the procedure. Assessing color (pale, cyanosis) and temperature (cool, cold) will identify a decrease in circulation. Once all peripheral and vascular assessment data are acquired, the primary health care provider would be notified. Simply documenting the findings is inappropriate. The leg would be positioned below the level of the heart to increase blood flow to the distal portion of the leg. Increasing intravenous fluids will not address the client's problem.

A nurse cares for a client with chronic obstructive pulmonary disease (COPD). The client states that going out with friends is no longer enjoyable. How would the nurse respond? a. "There are a variety of support groups for people who have COPD." b. "I will ask your primary health care provider to prescribe an antianxiety agent." c. "I'd like to hear about thoughts and feelings causing you to limit social activities." d. "Friends can be a good support system for clients with chronic disorders."

ANS: C Many clients with moderate to severe COPD become socially isolated because they are embarrassed by frequent coughing and mucus production. They also can experience fatigue, which limits their activities. The nurse needs to encourage the client to verbalize thoughts and feelings so that appropriate interventions can be selected. Joining a support group would not decrease feelings of social isolation if the client does not verbalize feelings. Antianxiety agents will not help the client with social isolation. While friends can be good sources of support, the client specifically is discussing going out of the home.

The primary health care provider requests the nurse start an infusion of milrinone on a client. How does the nurse explain the action of this drug to the client and spouse? a. "It constricts vessels, improving blood flow." b. "It dilates vessels, which lessens the work of the heart." c. "It increases the force of the heart's contractions." d. "It slows the heart rate down for better filling."

ANS: C Milrinone, is a positive inotrope, is a medication that increases the strength of the heart's contractions. It is not a vasoconstrictor, a vasodilator, nor does it slow the heart rate.

A nurse assesses clients on a medical-surgical unit. Which client would the nurse identify as having the greatest risk for cardiovascular disease? a. An 86-year-old man with a history of asthma. b. A 32-year-old man with colorectal cancer. c. A 65-year-old woman with diabetes mellitus. d. A 53-year-old postmenopausal woman who takes bisphosphonates.

ANS: C Of the options, the client with diabetes has a two- to four-fold increase in risk for death due to cardiovascular disease. Advancing age also increases risk, but not as much. Asthma, colorectal cancer, and bisphosphonate therapy do not increase the risk for cardiovascular disease.

A charge nurse is rounding on several older clients on ventilators in the Intensive Care Unit whom the nurse identifies as being at high risk for ventilator-associated pneumonia. To reduce this risk, what activity would the nurse delegate to the assistive personnel (AP)? a. Encourage between-meal snacks. b. Monitor temperature every 4 hours. c. Provide oral care every 4 hours. d. Report any new onset of cough.

ANS: C Oral colonization by gram-negative bacteria is a risk factor for health care-associated pneumonia. Good, frequent oral care can help prevent this from developing and is a task that can be delegated to the AP. Encouraging good nutrition is important, but this will not prevent pneumonia. Monitoring temperature and reporting new cough in clients are important to detect the onset of possible pneumonia but do not prevent it.

A nurse assesses an older adult client who has multiple chronic diseases. The client's heart rate is 48 beats/min. What action would the nurse take first? a. Document the finding in the chart. b. Initiate external pacing. c. Assess the client's medications. d. Administer 1 mg of atropine.

ANS: C Pacemaker cells in the conduction system decrease in number as a person ages, potentially resulting in bradycardia. However, the nurse would first check the medication reconciliation for medications that might cause such a drop in heart rate, and then would inform the primary health care provider. Documentation is important, but it is not the first action. The heart rate is not low enough for atropine or an external pacemaker to be needed unless the client is symptomatic, which is not apparent.

A client presents to the emergency department with an acute myocardial infarction (MI) at 15:00 (3:00 p.m.). The facility has 24-hour catheterization laboratory abilities. To improve client outcomes, by what time would the client have a percutaneous coronary intervention performed? a. 15:30 (3:30 p.m.) b. 16:00 (4:00 p.m.) c. 16:30 (4:30 p.m.) d. 17:00 (5:00 p.m.)

ANS: C Percutaneous coronary intervention would be performed within 90 minutes of diagnosis of myocardial infarction. Therefore, the client would have a percutaneous coronary intervention performed no later than 16:30 (4:30 p.m.).

A client had an acute myocardial infarction. What assessment finding indicates to the nurse that a significant complication has occurred? a. Blood pressure that is 20 mm Hg below baseline b. Oxygen saturation of 94% on room air c. Poor peripheral pulses and cool skin d. Urine output of 1.2 mL/kg/hr for 4 hours

ANS: C Poor peripheral pulses and cool skin may be signs of impending cardiogenic shock and would be reported immediately. A blood pressure drop of 20 mm Hg may not be worrisome. An oxygen saturation of 94% is just slightly below normal. A urine output of 1.2 mL/kg/hr for 4 hours is normal.

After teaching a client who is prescribed salmeterol, the nurse assesses the client's understanding. Which statement by the client indicates a need for additional teaching? a. "I will be certain to shake the inhaler well before I use it." b. "It may take a while before I notice a change in my asthma." c. "I will use the drug when I have an asthma attack." d. "I will be careful not to let the drug escape out of my nose and mouth."

ANS: C Salmeterol is a long-acting beta2 agonist designed to prevent an asthma attack; it does not relieve or reverse symptoms. Salmeterol has a slow onset of action; therefore, it would not be used as a rescue drug. The drug must be shaken well because it has a tendency to separate easily. Poor technique on the client's part allows the drug to escape through the nose and mouth.

A nurse cares for a client who has developed esophagitis after undergoing radiation therapy for lung cancer. Which diet selection would the nurse provide for this client? a. Spaghetti with meat sauce, ice cream b. Chicken soup, grilled cheese sandwich c. Omelet, soft whole-wheat bread d. Pasta salad, custard, orange juice

ANS: C Side effects of radiation therapy may include inflammation of the esophagus. Clients would be taught that bland, soft, high-calorie foods are best, along with liquid nutritional supplements. Tomato sauce may prove too spicy for a client with esophagitis. A grilled cheese sandwich is too difficult to swallow with this condition, and orange juice and other foods with citric acid are too caustic.

A nurse is assessing a client who is suspected of having ARDS. The nurse is confused that although the client appears dyspneic and the oxygen saturation is 88% on 6 L/min of oxygen, the client's lungs are clear. What explanation does the more senior nurse provide? a. "The client is too dehydrated for moist-sounding lungs." b. "The client hasn't started having any bronchospasm yet." c. "Lung edema is in the interstitial tissues, not the airways." d. "Clients with ARDS usually have clear lung sounds."

ANS: C The clear lung sounds are due to the fact that the edema is found in the lung interstitial tissues, where it can't be auscultated, instead of in the airways. It is not related to the client being dehydrated or having bronchospasm. The statement about all clients with ARDS having clear lung sounds does not provide any information.

A nurse assesses clients on the medical-surgical unit. Which client is at greatest risk for development of obstructive sleep apnea? a. A 26-year-old woman who is 8 months pregnant. b. A 42-year-old man with gastroesophageal reflux disease. c. A 55-year-old woman who is 50 lb (23 kg) overweight. d. A 73-year-old man with type 2 diabetes mellitus.

ANS: C The client at highest risk would be the one who is extremely overweight. None of the other clients have risk factors for sleep apnea. Clients with sleep apnea may develop gastroesophageal reflux.

A client with acute respiratory failure is on a ventilator and is sedated. What care may the nurse delegate to the assistive personnel AP)? a. Assess the client for sedation needs. b. Get family permission for restraints. c. Provide frequent oral care per protocol. d. Use nonverbal pain assessment tools.

ANS: C The client on mechanical ventilation needs frequent oral care, which can be delegated to the AP. The other actions fall within the scope of practice of the nurse.

A nurse cares for a client who is scheduled for a total laryngectomy. What action would the nurse take prior to surgery? a. Assess airway patency, breathing, and circulation. b. Administer prescribed intravenous pain medication. c. Assist the client to choose a communication method. d. Ambulate the client in the hallway to assess gait.

ANS: C The client will not be able to speak after surgery. The nurse would assist the client to choose a communication method that he or she would like to use after surgery. Assessing the patient's airway and administering IV pain medication are done after the procedure. Although ambulation promotes health and decreases the complications of any surgery, this patient's gait would not be impacted by a total laryngectomy and therefore is not a priority.

The nurse is caring for a client who has cystic fibrosis (CF). The client asks for information about gene therapy. What response by the nurse is best? a. "Unfortunately, gene therapy is only provided to children upon diagnosis." b. "Do you know that you will have to have genetic testing?" c. "There is a good treatment for the most common genetic defect in CF." d. "Gene therapy will only help improve your pulmonary symptoms."

ANS: C The drug ivacaftor/lumacaftor is effective as therapy for patients whose CF is caused by the F508del (also known as the Phe508del) mutation, the most common mutation involved in CF, even in patients who are homozygous for the mutation with both alleles being affected. The nurse would provide that information as the best response. Asking if the client understands he or she will have to undergo genetic testing is a correct statement, but is a yes/no question which is not therapeutic and might sound paternalistic. It also does not provide any information on the therapy itself. The drug is not limited to children and helps move chloride closer to the membrane surfaces so it would have an effect on any organ compromised by CF.

1A nurse prepares a client for cardiac catheterization. The client states, "I am afraid I might die." What is the nurse's best response? a. "This is a routine test and the risk of death is very low." b. "Would you like to speak with a chaplain prior to test?" c. "Tell me more about your concerns about the test." d. "What support systems do you have to assist you?"

ANS: C The nurse would discuss the client's feelings and concerns related to the cardiac catheterization. The nurse would not provide false hope or push the client's concerns off on the chaplain. The nurse would address support systems after addressing the client's current issue.

A nurse prepares a client for coronary artery bypass graft surgery. The client states, "I am afraid I might die." What is the nurse's best response? a. "This is a routine surgery and the risk of death is very low." b. "Would you like to speak with a chaplain prior to surgery?" c. "Tell me more about your concerns about the surgery." d. "What support systems do you have to assist you?"

ANS: C The nurse would discuss the client's feelings and concerns related to the surgery. The nurse would not provide false hope or simply call the chaplain. The nurse would address support systems after addressing the client's current issue.

A nurse is caring for a client who had a modified uvulopalatopharyngoplasty (modUPPP) earlier in the day for obstructive sleep apnea. Which assessment finding indicates that a priority goal has been met? a. Client reports pain is controlled satisfactorily with analgesic regime. b. Client does not have foul odor to the breath or beefy red mucus membranes. c. Client is able to swallow own secretions without drooling. d. Client's vital signs are within normal parameters.

ANS: C The priority after a modUPPP is maintaining a patent airway. The client who has a patent airway can swallow his or her own secretions without drooling. Controlled pain is important, but not the priority. Foul breath odor and beefy red mucus membranes indicate possible infection, which probably would not occur this soon after surgery, but preventing infection does not take priority over airway. Vital signs "within normal parameters" are vague.

A client has been brought to the emergency department with a life-threatening chest injury. What action by the nurse takes priority? a. Apply oxygen at 100%. b. Assess the respiratory rate. c. Ensure a patent airway. d. Start two large-bore IV lines.

ANS: C The priority for any chest trauma client is airway, breathing, and circulation. The nurse first ensures that the client has a patent airway. Assessing respiratory rate and applying oxygen are next, followed by inserting IVs.

A nurse cares for a client who had a bronchoscopy 2 hours ago. The client asks for a drink of water. What action would the nurse take next? a. Call the primary health care provider and request food and water for the client. b. Provide the client with ice chips instead of a drink of water. c. Assess the client's gag reflex before giving any food or water. d. Let the client have a small sip to see whether he or she can swallow.

ANS: C The topical anesthetic used during the procedure will have affected the client's gag reflex. Before allowing the client anything to eat or drink, the nurse must check for the return of this reflex.

A client is admitted with suspected pneumonia from the emergency department. The client went to the primary health care provider a "few days ago" and shows the nurse the results of what the client calls "an allergy test," as shown below: The reddened area is firm. What action by the nurse is best? a. Assess the client for possible items to which he or she is allergic. b. Call the primary health care provider's office to request records. c. Immediately place the client on Airborne Precautions. d. Prepare to begin administration of intravenous antibiotics.

ANS: C This "allergy test" is actually a positive tuberculosis test. The client would be placed on Airborne Precautions immediately. The other options do not take priority over preventing the spread of the disease.

A nurse in a family practice clinic is preparing discharge instructions for a client reporting facial pain that is worse when bending over, tenderness across the cheeks, and postnasal discharge. What instruction will be most helpful? a. "Ice packs may help with the facial pain." b. "Limit fluids to dry out your sinuses." c. "Try warm, moist heat packs on your face." d. "We will schedule a computed tomography scan this week."

ANS: C This client has rhinosinusitis. Comfort measures for this condition include humidification, hot packs, nasal saline irrigations, sleeping with the head elevated, increased fluids, and avoiding cigarette smoke. The client does not need a CT scan.

A nurse is providing tracheostomy care. What action by the nurse requires intervention by the charge nurse? a. Holding the device securely when changing ties b. Suctioning the client first if secretions are present c. Tying a square knot at the back of the neck d. Using half-strength peroxide for cleansing

ANS: C To prevent pressure injuries and for client safety, when ties are used that must be knotted, the knot would be placed at the side of the client's neck, not in back. The other actions are appropriate.

An intubated client's oxygen saturation has dropped to 88%. What action by the nurse takes priority? a. Determine if the tube is kinked. b. Ensure that all connections are patent. c. Listen to the client's lung sounds. d. Suction the endotracheal tube.

ANS: C When an intubated client shows signs of hypoxia, check for DOPE: displaced tube (most common cause), obstruction (often by secretions), pneumothorax, and equipment problems. The nurse listens for equal, bilateral breath sounds first to determine if the endotracheal tube is still correctly placed. If this assessment is normal, the nurse would follow the mnemonic and perform suction if needed, assess for pneumothorax, and finally check the equipment.

A nurse cares for a client after radiation therapy for neck cancer. The client reports extreme dry mouth. What action by the nurse is most appropriate? a. Ask the client to gargle with mouthwash containing lidocaine. b. Administer IV fluid boluses every 2 hours. c. Explain that xerostomia may be a permanent side effect. d. Assess the client's neck for redness and swelling.

ANS: C Xerostomia, or dry mouth, is a potential side effect of radiation, particularly if the salivary glands were in the radiation zone. Unfortunately, this may be long term or even permanent. Gargling with lidocaine would not help. Increasing fluids is somewhat helpful, but the client would be encouraged to drink. The client's neck may have redness and swelling, but this finding is not related to the reported dry mouth.

The emergency department nurse is participating in a bioterrorism drill in which several "clients" are suspected to have inhalation anthrax. Which "clients" would the nurse see as the priorities? (Select all that apply.) a. Widened mediastinum on chest x-ray b. Dry cough c. Stridor d. Oxygen saturation of 91% e. Diaphoresis f. Oral temperature of 99.9° F (37.7° C)

ANS: C, D, E Clients with fulminant anthrax may exhibit stridor, hypoxia, and diaphoresis. Although an oxygen saturation of 91% is not critical, it is abnormally low. These clients would be seen as the priority. A widened mediastinum and dry cough are usually seen in the prodromal phase when the temperature elevation is not as severe.

Which statements by the client indicate good understanding of foot care in peripheral vascular disease? (Select all that apply.) a. "A good abrasive pumice stone will keep my feet soft." b. "I'll always wear shoes if I can buy cheap flip-flops." c. "I will keep my feet dry, especially between the toes." d. "Lotion is important to keep my feet smooth and soft." e. "Washing my feet in room-temperature water is best." f. "I will inspect my feet daily."

ANS: C, D, E Good foot care includes appropriate hygiene and injury prevention. Keeping the feet dry; wearing good, comfortable shoes; using lotion; washing the feet in room-temperature water; cutting the nails straight across; and inspecting the feet daily are all important measures. Abrasive material such as pumice stones would not be used. Cheap flip-flops may not fit well and won't offer much protection against injury.

When working with women who are taking hormonal birth control, what health promotion measures does the nurse teach to prevent possible pulmonary embolism (PE)? (Select all that apply.) a. Avoid drinking alcohol. b. Eat more omega-3 fatty acids. c. Exercise on a regular basis. d. Maintain a healthy weight. e. Stop smoking cigarettes.

ANS: C, D, E Health promotion measures for clients to prevent thromboembolic events such as PE include maintaining a healthy weight, exercising on a regular basis, and not smoking. Avoiding alcohol and eating more foods containing omega-3 fatty acids are heart-healthy actions but do not relate to the prevention of PE.

A nurse assesses a client with asthma and notes bilateral wheezing, decreased pulse oxygen saturation, and suprasternal retraction on inhalation. What actions by the nurse are best? (Select all that apply.) a. Administer prescribed salmeterol inhaler. b. Assess the client for a tracheal deviation. c. Administer oxygen and place client on an oximeter. d. Perform peak expiratory flow readings. e. Administer prescribed albuterol inhaler. f. Assess the client's lung sounds after administering the inhaler.

ANS: C, E, F Suprasternal retraction caused by inhalation usually indicates that the client is using accessory muscles and is having difficulty moving air into the respiratory passages because of airway narrowing. Wheezing indicates a narrowed airway; a decreased pulse oxygen saturation also supports this finding. The asthma is becoming unstable, and intervention is needed. Administration of a rescue inhaler is indicated, probably along with administration of oxygen. The nurse would reassess the lung sounds after the rescue inhaler. The nurse would not do a peak flow reading at this time, nor would a code be called. The nurse could assess for tracheal deviation after administering oxygen and albuterol.

A nurse obtains the health history of a client who is recently diagnosed with lung cancer and identifies that the client has a 60-pack-year smoking history. Which action is most important for the nurse to take when interviewing this client? a. Tell the client that he or she needs to quit smoking to stop further cancer development. b. Encourage the client to be completely honest about both tobacco and marijuana use. c. Maintain a nonjudgmental attitude to avoid causing the client to feel guilty. d. Avoid giving the client false hope regarding cancer treatment and prognosis.

ANS: CSmoking assessments and cessation information can be an uncomfortable and sensitive topic among both clients and health care providers. The nurse would maintain a nonjudgmental attitude in order to foster trust with the client. Telling the client he or she needs to quit smoking is paternalistic and threatening. Assessing exposure to smoke includes more than tobacco and marijuana. The nurse would avoid giving the client false hope but when taking a history, it is most important to get accurate information.

A nurse cares for a client who had a chest tube placed 6 hours ago and refuses to take deep breaths because of the pain. What action would the nurse take? a. Ambulate the client in the hallway to promote deep breathing. b. Auscultate the client's anterior and posterior lung fields. c. Encourage the client to take shallow breaths to help with the pain. d. Administer pain medication and encourage the client to take deep breaths.

ANS: D A chest tube is placed in the pleural space and may be uncomfortable for a client. The nurse would provide pain medication to minimize discomfort and encourage the client to take deep breaths. The other responses do not address the client's discomfort and need to take deep breaths to prevent complications.

A nurse is assessing a client who has suffered a nasal fracture. Which assessment would the nurse perform first? a. Facial pain b. Vital signs c. Bone displacement d. Airway patency

ANS: D A patent airway is the priority. The nurse first would make sure that the airway is patent and then would determine whether the client is in pain and whether bone displacement or blood loss has occurred.

A nurse assesses a client after a thoracentesis. Which assessment finding warrants immediate action? a. The client rates pain as a 5/10 at the site of the procedure. b. A small amount of drainage from the site is noted. c. Pulse oximetry is 93% on 2 L of oxygen. d. The trachea is shifted toward the opposite side of the neck.

ANS: D A shift of central thoracic structures toward one side is a sign of a tension pneumothorax, which is a medical emergency. The other findings are normal or near normal. The nurse would report this finding immediately or call the Rapid Response Team.

A nurse is caring for a client who is scheduled to undergo a thoracentesis. Which intervention would the nurse complete prior to the procedure? a. Measure oxygen saturation before and after a 12-minute walk. b. Verify that the client understands all possible complications. c. Explain the procedure in detail to the client and the family. d. Validate that informed consent has been given by the client.

ANS: D A thoracentesis is an invasive procedure with many potentially serious complications. The nurse would ensure signed informed consent has been obtained. Verifying that the client understands complications and explaining the procedure to be performed will be done by the primary health care provider, not the nurse. Measurement of oxygen saturation before and after a 12-minute walk is not a procedure unique to a thoracentesis.

A new nurse is caring for a client with an abdominal aneurysm. What action by the new nurse requires the nurse's mentor to intervene? a. Assesses the client for back pain. b. Auscultates over abdominal bruit. c. Measures the abdominal girth. d. Palpates the abdomen in four quadrants.

ANS: D Abdominal aneurysms should never be palpated as this increases the risk of rupture. The nurse mentoring the new nurse would intervene when the new nurse attempts to do this. The other actions are appropriate.

An emergency department nurse triages clients who present with chest discomfort. Which client would the nurse plan to assess first? a. Client who describes pain as a dull ache. b. Client who reports moderate pain that is worse on inspiration. c. Client who reports cramping substernal pain. d. Client who describes intense squeezing pressure across the chest.

ANS: D All clients who have chest pain would be assessed more thoroughly. To determine which client would be seen first, the nurse must understand common differences in pain descriptions. Intense stabbing and viselike (squeezing) substernal pain or pressure that spreads through the client 's chest, arms, jaw, back, or neck are indicatives of a myocardial infarction. The nurse would plan to see this client first to prevent cardiac cell death. A dull ache, pain that gets worse with inspiration, and cramping pain are not usually associated with myocardial infarction.

A nurse assesses a client who is scheduled for a cardiac catheterization. Which assessment would the nurse complete as the priority prior to this procedure? a. Client's level of anxiety b. Ability to turn self in bed c. Cardiac rhythm and heart rate d. Allergies to iodine-based agents

ANS: D Before the procedure, the nurse would ascertain whether the client has an allergy to iodine-containing preparations, such as seafood or local anesthetics. The contrast medium used during the procedure is iodine based. This allergy can cause a life-threatening reaction, so it is a high priority. It is important for the nurse to assess anxiety, mobility, and baseline cardiac status, but allergies take priority for client safety.

A nurse assesses a client after administering a prescribed beta blocker. Which assessment would the nurse expect to find? a. Blood pressure increased from 98/42 to 132/60 mm Hg. b. Respiratory rate decreased from 25 to 14 breaths/min. c. Oxygen saturation increased from 88% to 96%. d. Pulse decreased from 100 to 80 beats/min.

ANS: D Beta blockers block the stimulation of beta1-adrenergic receptors. They block the sympathetic (fight-or-flight) response and decrease the heart rate (HR). The beta blocker will decrease HR and blood pressure, increasing ventricular filling time. It usually does not have effects on beta2-adrenergic receptor sites. Cardiac output may drop because of decreased HR, but slowing the rate may allow for better filling and better cardiac output.

An older client with peripheral vascular disease (PVD) is explaining the daily foot care regimen to the family practice clinic nurse. What statement by the client may indicate a barrier to proper foot care? a. "I nearly always wear comfy sweatpants and house shoes." b. "I'm glad I get energy assistance so my house isn't so cold." c. "My daughter makes sure I have plenty of lotion for my feet." d. "My hands shake when I try to do things requiring coordination."

ANS: D Clients with PVD need to pay special attention to their feet. Toenails need to be kept short and cut straight across. The client whose hands shake may cause injury when trimming toenails. The nurse would refer this client to a podiatrist. Comfy sweatpants and house shoes are generally loose and not restrictive, which is important for clients with PVD. Keeping the house at a comfortable temperature makes it less likely the client will use alternative heat sources, such as heating pads, to stay warm. The client should keep the feet moist and soft with lotion.

A nurse teaches a client who has a history of heart failure. Which statement would the nurse include in this client's discharge teaching? a. "Avoid drinking more than 3 quarts (3 L) of liquids each day." b. "Eat six small meals daily instead of three larger meals." c. "When you feel short of breath, take an additional diuretic." d. "Weigh yourself daily while wearing the same amount of clothing."

ANS: D Clients with heart failure are instructed to weigh themselves daily to detect worsening heart failure early, and thus avoid complications. Other signs of worsening heart failure include increasing dyspnea, exercise intolerance, cold symptoms, and nocturia. Fluid overload increases symptoms of heart failure. The client would be taught to eat a heart-healthy diet, balance intake and output to prevent dehydration and overload, and take medications as prescribed. The most important discharge teaching is daily weights as this provides the best data related to fluid retention.

A client is being discharged on long-term therapy for tuberculosis (TB). What referral by the nurse is most appropriate? a. Community social worker for Meals on Wheels b. Occupational therapy for job retraining c. Physical therapy for homebound therapy services d. Visiting nurses for directly observed therapy

ANS: D Directly observed therapy is often utilized for managing clients with TB in the community. Meals on Wheels, job retraining, and home therapy may or may not be appropriate.

A nurse cares for a client who has packing inserted for posterior nasal bleeding. What action would the nurse take first? a. Assess the client's pain level. b. Keep the client's head elevated. c. Teach the client about the causes of nasal bleeding. d. Assess the client's airway.

ANS: D If the packing slips out of place, it may obstruct the client's airway. The other options are good interventions, but ensuring that the airway is patent in the priority objective.

A client has been admitted for suspected inhalation anthrax infection. What question by the nurse is most important? a. "Are any family members also ill?" b. "Have you traveled recently?" c. "How long have you been ill?" d. "What is your occupation?"

ANS: D Inhalation anthrax is rare and is an occupational hazard among people who work with animal wool, bone meal, hides, and skin, such as taxidermists and veterinarians. Inhalation anthrax seen in someone without an occupational risk is considered a bioterrorism event and must be reported to authorities immediately. The other questions are appropriate for anyone with an infection.

1. A nurse assesses a client's respiratory status. Which information is most important for the nurse to obtain? a. Average daily fluid intake. b. Neck circumference. c. Height and weight. d. Occupation and hobbies.

ANS: D Many respiratory problems occur as a result of chronic exposure to inhalation irritants used in a client's occupation and hobbies. Although it will be important for the nurse to assess the client's fluid intake, height, and weight, these will not be as important as determining his occupation and hobbies. This is part of the I-PREPARE assessment model for particulate matter exposure. Determining the client's neck circumference will not be an important part of a respiratory assessment.

A nurse assesses a client's electrocardiograph tracing and observes that not all QRS complexes are preceded by a P wave. How would the nurse interpret this observation? a. The client has hyperkalemia causing irregular QRS complexes. b. Ventricular tachycardia is overriding the normal atrial rhythm. c. The client's chest leads are not making sufficient contact with the skin. d. Ventricular and atrial depolarizations are initiated from different sites.

ANS: D Normal rhythm shows one P wave preceding each QRS complex, indicating that all depolarization is initiated at the sinoatrial node. QRS complexes without a P wave indicate a different source of initiation of depolarization. This finding on an electrocardiograph tracing is not an indication of hyperkalemia, ventricular tachycardia, or disconnection of leads.

A client undergoing hemodynamic monitoring after a myocardial infarction has a right atrial pressure of 0.5 mm Hg. What action by the nurse is most appropriate? a. Level the transducer at the phlebostatic axis. b. Lay the client in the supine position. c. Prepare to administer diuretics. d. Prepare to administer a fluid bolus.

ANS: D Normal right atrial pressures are from 0 to 8 mm Hg. This pressure is at the extreme lower end, which indicates hypovolemia, so the nurse would prepare to administer a fluid bolus. The transducer would remain leveled at the phlebostatic axis. Positioning may or may not influence readings but a reading this low is definitive for volume depletion. Diuretics would be contraindicated.

15. A nurse evaluates the following arterial blood gas and vital sign results for a client with chronic obstructive pulmonary disease (COPD): Arterial Blood Gas Results pH = 7.32 PaCO2 = 62 mm Hg PaO2 = 46 mm Hg HCO3 = 28 mEq/L (28 mmol/L) Vitals Heart rate = 110 beats/min Respiratory rate = 12 breaths/min Blood pressure = 145/65 mm Hg Oxygen saturation = 76% What action would the nurse take first? a. Administer a short-acting beta2 agonist inhaler. b. Document the findings as normal for a client with COPD. c. Teach the client diaphragmatic breathing techniques. d. Initiate oxygenation therapy to increase saturation to 88% to 92%.

ANS: D Oxygen would be administered to a client who is hypoxic even if the client has COPD and is a carbon dioxide retainer. The other interventions do not address the client's hypoxia, which is the major issue. There is no indication the client needs an inhaler. Diaphragmatic breathing techniques would not be taught to a client in distress. These findings are not normal for all clients with COPD.

A nurse cares for a client who has a pleural chest tube. What action would the nurse take to ensure safe use of this equipment? a. Strip the tubing to minimize clot formation and ensure patency. b. Secure tubing junctions with clamps to prevent accidental disconnections. c. Connect the chest tube to wall suction as prescribed by the primary health care provider. d. Keep padded clamps at the bedside for use if the drainage system is interrupted.

ANS: D Padded clamps would be kept at the bedside for use if the drainage system becomes dislodged or is interrupted. The nurse would never strip the tubing. Tubing junctions would be taped, not clamped. Wall suction would be set at the level indicated by the device's manufacturer, not the primary health care provider.

A nurse is caring for a client with acute pericarditis who reports substernal precordial pain that radiates to the left side of the neck. Which nonpharmacologic comfort measure would the nurse implement? a. Apply an ice pack to the client's chest. b. Provide a neck rub, especially on the left side. c. Allow the client to lie in bed with the lights down. d. Sit the client up with a pillow to lean forward on.

ANS: D Pain from acute pericarditis may worsen when the client lays supine. The nurse would position the client in a comfortable position, which usually is upright and leaning slightly forward. An ice pack and neck rub will not relieve this pain. Dimming the lights will also not help the pain.

A client is in the preoperative holding area prior to an emergency coronary artery bypass graft (CABG). The client is yelling at family members and tells the doctor to "just get this over with" when asked to sign the consent form. What action by the nurse is best? a. Ask the family members to wait in the waiting area. b. Inform the client that this behavior is unacceptable. c. Stay out of the room to decrease the client's stress levels. d. Tell the client that anxiety is common and that you can help.

ANS: D Preoperative fear and anxiety are common prior to cardiac surgery, especially in emergent situations. The client is exhibiting anxiety, and the nurse would reassure the client that fear is common and offer to help. The other actions will not reduce the client's anxiety.

A new nurse asks for an explanation of "refractory hypoxemia." What answer by the staff development nurse is best? a. "It is chronic hypoxemia that accompanies restrictive airway disease." b. "It is hypoxemia from lung damage due to mechanical ventilation." c. "It is hypoxemia that continues even after the client is weaned from oxygen." d. "It is hypoxemia that persists even with 100% oxygen administration."

ANS: D Refractory hypoxemia is hypoxemia that persists even with the administration of 100% oxygen. It is a cardinal sign of acute respiratory distress syndrome. It does not accompany restrictive airway disease and is not caused by the use of mechanical ventilation or by being weaned from oxygen.

A client is in the family practice clinic reporting a severe "cold" that started 4 days ago. On examination, the nurse notes that the client also has a severe headache and muscle aches. What action by the nurse is best? a. Educate the client on oseltamivir. b. Facilitate admission to the hospital. c. Instruct the client to have a flu vaccine. d. Teach the client to sneeze in the upper sleeve.

ANS: D Sneezing and coughing into one's sleeve helps prevent the spread of upper respiratory infections. The client does have symptoms of the flu (influenza), but it is too late to start antiviral medications; to be effective, they must be started within 24 to 48 hours of symptom onset. The client does not need hospital admission. The client would be instructed to have a flu vaccination, but now that he or she has the flu, vaccination will have to wait until next year.

A client is on mechanical ventilation and the client's spouse wonders why ranitidine is needed since the client "only has lung problems." What response by the nurse is best? a. "It will increase the motility of the gastrointestinal tract." b. "It will keep the gastrointestinal tract functioning normally." c. "It will prepare the gastrointestinal tract for enteral feedings." d. "It will prevent ulcers from the stress of mechanical ventilation."

ANS: D Stress ulcers can occur in many clients who are receiving mechanical ventilation, and often prophylactic medications are used to prevent them and possible subsequent aspiration. Frequently used medications include antacids, histamine blockers, and proton pump inhibitors. Ranitidine is a histamine-blocking agent.

A nurse assesses several clients who have a history of respiratory disorders. Which client would the nurse assess first? a. A 66-year-old client with a barrel chest and clubbed fingernails b. A 48-year-old client with an oxygen saturation level of 92% at rest c. A 35-year-old client who reports orthopnea in bed d. A 27-year-old client with a heart rate of 120 beats/min

ANS: D Tachycardia can indicate hypoxemia as the body tries to circulate the oxygen that is available. A barrel chest is not an emergency finding. Likewise, a pulse oximetry level of 92% is not considered an acute finding. Orthopnea at night in bed is breathlessness when lying down but is not an acute finding at this moment.

A client has progressed to Killip class III heart failure after a myocardial infarction. What does the nurse anticipate the client's care to include? a. Diuretics b. Nitrates c. Clopidogrel d. Dobutamine

ANS: D The client in class III heart failure would benefit from a positive inotrope such as dobutamine. Clients in class I typically respond well to diuretics and nitrates so this client would already be on these medications. Clopidogrel is a platelet inhibitor that will be prescribed for anyone having acute coronary syndrome for at least 12 months.

A nurse cares for a client who has advanced cardiac disease and states, "I am having trouble breathing while I'm sleeping at night." What is the nurse's best response? a. "I will consult your primary health care provider to prescribe a sleep study." b. "You become hypoxic while sleeping; oxygen therapy via nasal cannula will help." c. "A continuous positive airway pressure, or CPAP, breathing mask will help you breathe at night." d. "Use pillows to elevate your head and chest while you are sleeping."

ANS: D The client is experiencing orthopnea (shortness of breath while lying flat). The nurse would teach the client to elevate the head and chest with pillows or sleep in a recliner. A sleep study is not necessary to diagnose this client. Oxygen and CPAP will not help a client with orthopnea.

A nurse cares for a client with end-stage heart failure who is awaiting a transplant. The client appears depressed and states, "I know a transplant is my last chance, but I don't want to become a vegetable." How would the nurse respond? a. "Would you like to speak with a priest or chaplain?" b. "I will arrange for a psychiatrist to speak with you." c. "Do you want to come off the transplant list?" d. "Would you like information about advance directives?"

ANS: D The client is verbalizing a real concern or fear about negative outcomes of the surgery. This anxiety itself can have a negative effect on the outcome of the surgery because of sympathetic stimulation. The best action is to allow the client to verbalize the concern and work toward a positive outcome without making the client feel as though the concerns are not valid. The client needs to feel that he or she has some control over the future. The nurse personally provides care to address the client's concerns instead of immediately calling for the chaplain or psychiatrist. The nurse would not jump to conclusions and suggest taking the client off the transplant list, which is the best treatment option.

A nurse is assessing the peripheral vascular system of an older adult. What action by the nurse would cause the supervising nurse to intervene? a. Assessing blood pressure in both upper extremities b. Auscultating the carotid arteries for any bruits c. Classifying capillary filling of 4 seconds as normal d. Palpating both carotid arteries at the same time

ANS: D The nurse would not compress both carotid arteries at the same time to avoid brain ischemia. Blood pressure would be taken and compared in both arms. Prolonged capillary filling is considered to be greater than 5 seconds in an older adult, so classifying refill of 4 seconds as normal would not require intervention. Bruits would be auscultated.

A nurse is caring for a client on mechanical ventilation. When double-checking the ventilator settings with the respiratory therapist, what would the nurse ensure? a. The client is able to initiate spontaneous breaths. b. The inspired oxygen has adequate humidification. c. The upper peak airway pressure limit alarm is off. d. The upper peak airway pressure limit alarm is on.

ANS: D The upper peak airway pressure limit alarm will sound when the airway pressure reaches a preset maximum. This is critical to prevent barotrauma to the lungs. Alarms are never be turned off. Initiating spontaneous breathing is important for some modes of ventilation but not others. Adequate humidification is important but does not take priority over preventing injury.

A nurse assesses a client after administering the first dose of a nitrate. The client reports a headache. What action would the nurse take? a. Initiate oxygen therapy. b. Hold the next dose. c. Instruct the client to drink water. d. Administer PRN acetaminophen.

ANS: D The vasodilating effects of nitrates frequently cause clients to have headaches during the initial period of therapy. The nurse would inform the client about this side effect and offer a mild analgesic, such as acetaminophen. The client's headache is not related to hypoxia or dehydration; therefore, applying oxygen and drinking water would not help. The client needs to take the medication as prescribed to prevent angina; the medication would not be held.

A client is brought to the emergency department after sustaining injuries in a severe car crash. The client's chest wall does not appear to be moving normally with respirations, oxygen saturation is 82%, and the client is cyanotic. What action does the nurse take first? a. Administer oxygen and reassess. b. Auscultate the client's lung sounds. c. Facilitate a portable chest x-ray. d. Prepare to assist with intubation.

ANS: D This client has signs and symptoms of flail chest and, with the other signs, needs to be intubated and mechanically ventilated immediately. The nurse does not have time to administer oxygen and wait to reassess, or to listen to lung sounds. A chest x-ray will be taken after the client is intubated.

A nurse is assessing a client who has a tracheostomy. The nurse notes that the tracheostomy tube is pulsing with the heartbeat as the client's pulse is being taken. No other abnormal findings are noted. What action by the nurse is most appropriate? a. Call the operating room to inform them of a pending emergency case. b. No action is needed at this time; this is a normal finding in some clients. c. Remove the tracheostomy tube; ventilate the client with a bag-valve-mask. d. Stay with the client and have someone else call the primary health care provider immediately.

ANS: D This client may have a tracheoinnominate artery fistula, which can be a life-threatening emergency if the artery is breached and the client begins to hemorrhage. Since no bleeding is yet present, the nurse stays with the client and asks someone else to notify the primary health care provider. If the client begins hemorrhaging, the nurse removes the tracheostomy and applies pressure at the bleeding site. The client will need to be prepared for surgery.

A nurse prepares to defibrillate a client who is in ventricular fibrillation. Which intervention is appropriate for the nurse to perform prior to defibrillating this client? a. Make sure that the defibrillator is set to the synchronous mode. b. Administer 1 mg of intravenous epinephrine. c. Test the equipment by delivering a smaller shock at 100 J. d. Ensure that everyone is clear of contact with the client and the bed.

ANS: D To avoid injury, the rescuer commands that all personnel clear contact with the client or the bed and ensures their compliance before delivery of the shock. Defibrillation is done in asynchronous mode. Equipment would not be tested before a client is defibrillated because this is an emergency procedure; equipment would be checked on a routine basis. Defibrillation takes priority over any medications.

The nurse is preparing to change a client's sternal dressing. What action by the nurse is most important? a. Assess vital signs. b. Don a mask and gown. c. Gather needed supplies. d. Perform hand hygiene.

ANS: D To prevent a sternal wound infection, the nurse washes hands or performs hand hygiene as a priority and uses sterile technique when changing the dressing. Vital signs do not necessarily need to be assessed beforehand. A mask and gown are not needed. The nurse would gather needed supplies, but this is not the priority.

A nurse is preparing a client for a femoropopliteal bypass operation. What actions does the nurse delegate to the assistive personnel (AP)? (Select all that apply.) a. Administering preoperative medication b. Ensuring that the consent is signed c. Marking pulses with a pen d. Raising the side rails on the bed e. Recording baseline vital signs

ANS: D, E The AP can raise the side rails of the bed for client safety and take and record the vital signs. Administering medications, ensuring that a consent is on the chart, and marking the pulses for later comparison would be done by the registered nurse. This is also often done by the postanesthesia care nurse and is part of the hand-off report.

most at risk for dig toxicity

anyone of potassium wasting diuretics - furosemide - thiazide renal impairment

pt in heart failure is constipated, what should the nurse recommend

walking increase fiber stool softeners no drinking excess fluid


Set pelajaran terkait

§9. Строение Земли

View Set

Modern And Contemporary Art Final

View Set

The Client with an Ectopic Pregnancy

View Set

Health Care Systems in UK and Canada

View Set

Grade 7 Unit 1- The Practice of Science (B)

View Set

AP GOPO Final (Reading Quiz Questions)

View Set

CASP Chapter 3 Risk Mitigation, Strategies, and Controls

View Set